You are on page 1of 97

Contents

1 International Mathematics Olympiad 6


1.1 1
st
IMO, Romania, 1959 . . . . . . . . . . . . . . . . . . . . . . . . . . . . 6
1.2 2
nd
IMO, Romania, 1960 . . . . . . . . . . . . . . . . . . . . . . . . . . . . 7
1.3 3
rd
IMO, Hungary, 1961 . . . . . . . . . . . . . . . . . . . . . . . . . . . . 8
1.4 4
th
IMO, Czechoslovakia, 1962 . . . . . . . . . . . . . . . . . . . . . . . . . 9
1.5 5
th
IMO, Poland, 1963 . . . . . . . . . . . . . . . . . . . . . . . . . . . . . 10
1.6 6
th
IMO, USSR, 1964 . . . . . . . . . . . . . . . . . . . . . . . . . . . . . . 11
1.7 7
th
IMO, West Germany, 1965 . . . . . . . . . . . . . . . . . . . . . . . . . 11
1.8 8
th
IMO, Bulgaria, 1966 . . . . . . . . . . . . . . . . . . . . . . . . . . . . 12
1.9 9
th
IMO, Yugoslavia, 1967 . . . . . . . . . . . . . . . . . . . . . . . . . . . 13
1.10 10
th
IMO, USSR, 1968 . . . . . . . . . . . . . . . . . . . . . . . . . . . . . 14
1.11 11
th
IMO, Romania, 1969 . . . . . . . . . . . . . . . . . . . . . . . . . . . 16
1.12 12
th
IMO, Hungary, 1970 . . . . . . . . . . . . . . . . . . . . . . . . . . . . 16
1.13 13
th
IMO, Czechoslovakia, 1971 . . . . . . . . . . . . . . . . . . . . . . . . 18
1
2 CONTENTS
1.14 14
th
IMO, USSR, 1972 . . . . . . . . . . . . . . . . . . . . . . . . . . . . . 18
1.15 15
th
IMO, USSR, 1973 . . . . . . . . . . . . . . . . . . . . . . . . . . . . . 19
1.16 16
th
IMO, West Germany, 1974 . . . . . . . . . . . . . . . . . . . . . . . . 20
1.17 17
th
IMO, Bulgaria, 1975 . . . . . . . . . . . . . . . . . . . . . . . . . . . . 21
1.18 18
th
IMO, Austria, 1976 . . . . . . . . . . . . . . . . . . . . . . . . . . . . 22
1.19 19
th
IMO, Yugoslavia, 1977 . . . . . . . . . . . . . . . . . . . . . . . . . . 23
1.20 20
th
IMO, Romania, 1978 . . . . . . . . . . . . . . . . . . . . . . . . . . . 24
1.21 21
st
IMO, United Kingdom, 1979 . . . . . . . . . . . . . . . . . . . . . . . 25
1.22 22
nd
IMO, Washington, USA, 1981 . . . . . . . . . . . . . . . . . . . . . . 26
1.23 23
rd
IMO, Budapest, Hungary, 1982 . . . . . . . . . . . . . . . . . . . . . . 27
1.24 24
th
IMO, Paris, France, 1983 . . . . . . . . . . . . . . . . . . . . . . . . . 28
1.25 25
th
IMO, Prague, Czechoslovakia, 1984 . . . . . . . . . . . . . . . . . . . 29
1.26 26
th
IMO, Helsinki, Finland, 1985 . . . . . . . . . . . . . . . . . . . . . . . 30
1.27 27
th
IMO, Warsaw, Poland, 1986 . . . . . . . . . . . . . . . . . . . . . . . 31
1.28 28
th
IMO, Havana, Cuba , 1987 . . . . . . . . . . . . . . . . . . . . . . . . 32
1.29 29
th
IMO, Camberra, Australia, 1988 . . . . . . . . . . . . . . . . . . . . . 33
1.30 30
th
IMO, Braunschweig, West Germany, 1989 . . . . . . . . . . . . . . . . 34
1.31 31
st
IMO, Beijing, Peoples Republic of China, 1990 . . . . . . . . . . . . . 36
1.32 32
nd
IMO, Sigtuna, Sweden, 1991 . . . . . . . . . . . . . . . . . . . . . . . 37
1.33 33
rd
IMO, Moscow, Russia, 1992 . . . . . . . . . . . . . . . . . . . . . . . . 38
CONTENTS 3
1.34 34
th
IMO, Istambul, Turkey, 1993 . . . . . . . . . . . . . . . . . . . . . . . 39
1.35 35
th
IMO, Hong Kong, 1994 . . . . . . . . . . . . . . . . . . . . . . . . . . 40
1.36 36
th
IMO, Toronto, Canada, 1995 . . . . . . . . . . . . . . . . . . . . . . . 41
1.37 37
th
IMO, Mumbai, India, 1996 . . . . . . . . . . . . . . . . . . . . . . . . 42
1.38 38
th
IMO, Mar del Plata, Argentina, 1997 . . . . . . . . . . . . . . . . . . 43
1.39 39
th
IMO, Taipei, Taiwan, 1998 . . . . . . . . . . . . . . . . . . . . . . . . 44
1.40 40
th
IMO, Bucharest, Romania, 1999 . . . . . . . . . . . . . . . . . . . . . 45
1.41 41
st
IMO, Taejon, South Korea, 2000 . . . . . . . . . . . . . . . . . . . . . 46
1.42 42
nd
IMO, Washington DC, USA, 2001 . . . . . . . . . . . . . . . . . . . . 47
1.43 43
rd
IMO, Glascow, United Kingdom, 2002 . . . . . . . . . . . . . . . . . . 48
1.44 44
th
IMO, Tokyo, Japan, 2003 . . . . . . . . . . . . . . . . . . . . . . . . . 49
2 William Lowell Putnam Competition 50
2.1 46
th
Anual William Lowell Putnam Competition, 1985 . . . . . . . . . . . 50
2.2 47
th
Anual William Lowell Putnam Competition, 1986 . . . . . . . . . . . 52
2.3 48
th
Anual William Lowell Putnam Competition, 1987 . . . . . . . . . . . 54
2.4 49
th
Anual William Lowell Putnam Competition, 1988 . . . . . . . . . . . 56
2.5 50
th
Anual William Lowell Putnam Competition, 1989 . . . . . . . . . . . 58
2.6 51
th
Anual William Lowell Putnam Competition, 1990 . . . . . . . . . . . 60
2.7 52
th
Anual William Lowell Putnam Competition, 1991 . . . . . . . . . . . 62
2.8 53
th
Anual William Lowell Putnam Competition, 1992 . . . . . . . . . . . 64
4 CONTENTS
2.9 54
th
Anual William Lowell Putnam Competition, 1993 . . . . . . . . . . . 66
2.10 55
th
Anual William Lowell Putnam Competition, 1994 . . . . . . . . . . . 68
2.11 56
th
Anual William Lowell Putnam Competition, 1995 . . . . . . . . . . . 69
2.12 57
th
Anual William Lowell Putnam Competition, 1996 . . . . . . . . . . . 71
2.13 58
th
Anual William Lowell Putnam Competition, 1997 . . . . . . . . . . . 73
2.14 59
th
Anual William Lowell Putnam Competition, 1998 . . . . . . . . . . . 75
2.15 60
th
Anual William Lowell Putnam Competition, 1999 . . . . . . . . . . . 76
2.16 61
st
Anual William Lowell Putnam Competition, 2000 . . . . . . . . . . . . 79
2.17 62
nd
Anual William Lowell Putnam Competition, 2001 . . . . . . . . . . . 80
2.18 63
rd
Anual William Lowell Putnam Competition, 2002 . . . . . . . . . . . 81
3 Asiatic Pacic Mathematical Olympiads 84
3.1 1
st
Asiatic Pacic Mathematical Olympiad, 1989 . . . . . . . . . . . . . . . 84
3.2 2
nd
Asiatic Pacic Mathematical Olympiad, 1990 . . . . . . . . . . . . . . 85
3.3 3
rd
Asiatic Pacic Mathematical Olympiad, 1991 . . . . . . . . . . . . . . 86
3.4 4
th
Asiatic Pacic Mathematical Olympiad, 1992 . . . . . . . . . . . . . . . 86
3.5 5
th
Asiatic Pacic Mathematical Olympiad, 1993 . . . . . . . . . . . . . . . 87
3.6 6
th
Asiatic Pacic Mathematical Olympiad, 1994 . . . . . . . . . . . . . . . 88
3.7 7
th
Asiatic Pacic Mathematical Olympiad, 1995 . . . . . . . . . . . . . . . 89
3.8 8
th
Asiatic Pacic Mathematical Olympiad, 1996 . . . . . . . . . . . . . . . 90
3.9 9
th
Asiatic Pacic Mathematical Olympiad, 1997 . . . . . . . . . . . . . . . 91
CONTENTS 5
3.10 10
th
Asiatic Pacic Mathematical Olympiad, 1998 . . . . . . . . . . . . . . 92
3.11 11
th
Asiatic Pacic Mathematical Olympiad, 1999 . . . . . . . . . . . . . . 93
3.12 12
th
Asiatic Pacic Mathematical Olympiad, 2000 . . . . . . . . . . . . . . 93
3.13 13
th
Asiatic Pacic Mathematical Olympiad, 2001 . . . . . . . . . . . . . . 94
3.14 14
th
Asiatic Pacic Mathematical Olympiad, 2002 . . . . . . . . . . . . . . 95
3.15 15
th
Asiatic Pacic Mathematical Olympiad, 2003 . . . . . . . . . . . . . . 96
Chapter 1
International Mathematics Olympiad
1.1 1
st
IMO, Romania, 1959
1. Prove that the fraction
21n + 4
14n + 3
is irreductible for every natural number n
2. For what real values of x is
_
x +

2x 1 +
_
x

2x 1 = A
given (a) A =

2, (b) A = 1, (c) A = 2, where only non-negative real numbers are


admitted for square roots?
3. Let a, b, c be real numbers. Consider the quadratic equation in cos x: a cos
2
x +
b cos x + c = 0. Using the numbers a, b and c, form a quadratic ecuation in cos 2x,
whose roots are the same as those of the original ecuation. Compare the ecuations
in cos x and cos 2x for a = 4, b = 2 and c = 1
4. Construct a right triangle with hypotenuse c such that the median drawn to the
hypotenuse is the geometric mean of the two legs of the triangle.
5. An arbitrary point M is selected in the interior of the segment AB. The squares
AMCD and MBEF are constructed on the same side of AB, sith the segments AM
and MB as their respective bases. The circles circumscribed abut these squares,
6
1.2. 2
ND
IMO, ROMANIA, 1960 7
with centers P and Q intersect at M and also at another point N. Let N

denote the
intersection of the straight lines AF and BC.
(a) Prove that the points N and N

coinside.
(b) Prove that the straight lines MN pass throught a xed point S independent of
the choice of M.
(c) Find the locus of the midpoints of the the segment PQ as M varies between A
and B.
6. Two planes, P and Q, intersect along the line p. The point A is given in the plane
P, and the point C in the plane Q; neither of these points lies on the straight line p.
Construct an isosceles trapezoid ABCD (with AB parallel to CD) in which a circle
can be inscribed, and with vertices B and D lying in the planes P and Q respectively.
1.2 2
nd
IMO, Romania, 1960
1. Determine all three-digit numbers N having the property that N is divisible by 11,
and
N
11
is equal to the sum of the squares of the digits of N.
2. For what values of the variable x does the following inequality hold?
4x
2
_
1

1 + 2x
_
2
< 2x + 9
3. In a given right triangle ABC, the hypotenuse BC, of lenght a, is dividen into n
equal parts (n an odd integer). Let be the acute angle subtending, from A, that
segment which contains the middle point of the hypotenuse. Let h be the lenght of
the altitude to the hypotenuse of the triangle. Prove:
tan =
4nh
(n
2
1) a
4. Construct a triangle ABC, given h
a
, h
b
(the altitudes fron A and B) and m
a
, the
median from vertex A.
5. Consider the cube ABCDA

(whith face ABCD directly above face A

).
(a) Find the locus of the midpoints of segment XY , where X is any point of AC
and Y is any point of B

.
8 CHAPTER 1. INTERNATIONAL MATHEMATICS OLYMPIAD
(b) Find the locus of points Z which lie on the segment XY of part (a) with ZY =
2XZ.
6. Considere a cone of revolution with an inscribed sphere tangent to the base of the
cone. A cylinder is circumscribed about this sphere so that one of its bases lies in
the base of the cone. Let V
1
be the volume of the cone and V
2
the volumen of the
cilinder.
(a) Prove that V
1
,= V
2
.
(b) Find the smallest number k for which V
1
= kV
2
, for this case, construct the
angle subtended by a diameter of the base of the cone at the vertex of the cone.
7. An isosceles trapezoid with bases a and c, and altitude h is given.
(a) On the axis of symmetry of this trapezoid, nd all points P such that both legs
of the trapezoid subtended right angles at P.
(b) Calculate the distance of P from either base.
(c) Determine under what conditions such points P actually exist. (Discuss varius
case that might arise)
1.3 3
rd
IMO, Hungary, 1961
1. Solve the system of equations:
x +y +z = a
x
2
+y
2
+z
2
= b
2
xy = z
2
where a and b are constants. Give the conditions that a and b must satisfy so that
x, y, z (the solutions of the system) are distinct positive numbers.
2. Let a, b, c the sides of a triangle, and T its area. Prove: a
2
+ b
2
+ c
2
4

3T. In
what case does the equality hold?
3. Solve the equation cos
n
x sin
n
x = 1, where n is a natural number.
4. Consider the triangle P
1
P
2
P
3
and a point P within the triangle. Lines PP
1
, PP
2
,
PP
3
intersect the opposite side in points Q
1
, Q
2
, Q
3
respectively. Prove that, of the
numbers
P
1
P
PQ
1
,
P
2
P
PQ
2
,
P
3
P
PQ
3
at least one is less than or equal to 2 and at least one is
grater than or equal to 2.
1.4. 4
TH
IMO, CZECHOSLOVAKIA, 1962 9
5. Construct triangle ABC if AC = b, AB = c and AMB = , where M is the
midpoint of the segment BC and < 90

. Prove that a solution exists and only if


b tan

2
c < b. In what case does the equality hold?
6. Considere a plane and three non-collinear points A, B, C on the same side of ;
suppose the plane determined by these three points is not parallel to . In plane a take
three arbitrary points A

, B

, C

. Let L, M, N be the midpoints of segments AA

,
BB

, CC

; let G the centroid of triangle LMN (We will not considere positions of
A

, B

, C

such that the points L, M, N do not form a triangle) What is the locus of
point G as A

, B

, C

range independently over the plane ?


1.4 4
th
IMO, Czechoslovakia, 1962
1. Find the smallest natural number n which has the following properties:
(a) Its decimal representation has 6 as the last digit.
(b) If the last digit 6 is erased and placed in front of the remaining digits, the
resulting number is four times as large as the original number n
2. Determine all real number x which satisfy the inequality:

3 x

x + 1 >
1
2
3. Consider the cube ABCDA

(ABCD and A

are the upper and lower


bases, respectively, and edges AA

, BB

, CC

, DD

are parallel) The point X moves


at constant speed along the perimeterof the square ABCD in the direction ABCDA,
and the point Y moves at the same rate along the perimeter of the square B

CB
in the direction B

CBB

. Points X and Y begin their motion at the same instant


from the starting position A and B

, respectively. Determine and draw the locus of


the midpoints of the segment XY .
4. Solve the ecuation cos
2
x + cos
2
2x + cos
2
3x = 1
5. On the circle K there are given three distinct points A, B, C. Construct (using only
straightedge and compasses) a fourth point D on K such that a circle can be inscribed
in the cuadrilateral thus obtained.
10 CHAPTER 1. INTERNATIONAL MATHEMATICS OLYMPIAD
6. Considere an isosceles triangle. let r be the radius of its circumscribed circle and
the radius of its inscribed circle. Prove that the distance d between the centers of
these two circles is
d =
_
r (r 2)
7. The tetrahedon SABC has the following propoerty: there exists ve spheres, each
tangent to the edges SA, SB, SC, BC, CA, AB or their extentions.
(a) Prove that the tetrahedron SABC is regular.
(b) Prove conversely that for every regular tetrahedron ve such spheres exist.
1.5 5
th
IMO, Poland, 1963
1. Find all real roots of the equation

x
2
p +2

x
2
1 = x, where p is a real param-
eter.
2. Point A and segment BC are given. Determine the locus of points in space which
are vertices of right angles with one side passing throught A, and the other side
intersecting the segment BC.
3. In an ngon all of whose interior angles are equal, the lenght of consecutive sides
satisfy the relation a
1
a
2
a
n
. Prove that a
1
= a
2
= = a
n
.
4. Find all solution x
1
, x
2
, x
3
, x
4
, x
5
of the system
(1) x
5
+x
2
= yx
1
(2) x
1
+x
3
= yx
2
(3) x
2
+x
4
= yx
3
(4) x
3
+x
5
= yx
4
(5) x
4
+x
1
= yx
5
where y is a parameter
5. Prove that
cos

7
cos
2
7
+ cos
3
7
=
1
2
6. Five students, A, B, C, D, E, took part in a contest. One prediction was that
contestants would nish in the order ABCDE. This prediction was very poor. In fact
no contestant nished in the position predicted, and no two contestants predicted to
nish consecutively actually did so. A second prediction has the contestants nishing
1.6. 6
TH
IMO, USSR, 1964 11
in the order DAECB. This prediction was better. Exactly two of the contestants
nished in the places predicted, and two disjoint pairs of students predicted to nish
consecutively actually did so. Determine the order in which the contestants nished.
1.6 6
th
IMO, USSR, 1964
1. (a) Find all positive integers n for which 2
n
1 is divisible by 7.
(b) Prove that there is not positive integer n such that 2
n
+ 1 is dibisible by 7.
2. Let a, b, c be the sides of a triangle. Prove that
a
2
(b +c a) +b
2
(c +a b) +c
2
(a +b c) 3abc
3. A circle is inscribed in triangle ABC with sides a, b, c. Tangents to the circle
parallel to the sides of the triangle are constructed. Each of these tangents cuts o a
triangle from ABC. In each of these triangle, a circle is inscribed. Find the sum
of the areas of all four inscribed circles (in terms of a, b, c)
4. Seventeen people correspond by mail with one another, each one with all the rest.
In their letters only three dierent topics are discussed. Each pair of correspondent
deals with only one of these topics. Prove that there are at least three people who
write to each other about the same topic.
5. Suppose ve points in a plane are situated so that no two of the straight lines joining
the other four points. Determine the maximum number of intersections that these
perpendiculars can have.
6. In tetrahedron ABCD, vertex D is connected with D
0
the centroid of ABC. Lines
parallel to DD
0
are drawn through A, B and C. These lines intersect the planes
BCD, CAD and ABD in points A
1
, B
1
and C
1
, respectively. Prove that the volume
of ABCD is one third the volume of A
1
B
1
C
1
D
0
. Is the result true if point D
0
is
selected anywhere within ABC?
1.7 7
th
IMO, West Germany, 1965
1. Determine all value x in the interval 0 x 2 which satisfy the inequality
2 cos x

1 + sin 2x

1 sin 2x

2
12 CHAPTER 1. INTERNATIONAL MATHEMATICS OLYMPIAD
2. Consider the system of equations
a
11
x
1
+a
12
x
2
+a
13
x
3
= 0
a
21
x
1
+a
22
x
2
+a
23
x
3
= 0
a
31
x
1
+a
32
x
2
+a
33
x
3
= 0
with unknowns x
1
, x
2
, x
3
. The coecient satisfy the conditions:
(a) a
11
, a
22
, a
33
are positive numbers;
(b) the remaining coecients are negative numbers;
(c) in each equation, the sum of the coecient is positive .
Prove that the given system has only the solution x
1
= x
2
= x
3
= 0.
3. Given the tetrahedron ABCD whose edges AB and CD have lenght a and b respec-
tively. The distance between the skew lines AB and CD is d, and the angle between
them is . Tetrahedron ABCD is divided into two solid by plane , parallel to lines
AB and CD. The ratio of the distances of from AB and CD is equal to k. Compute
the ratio of the volumes of the two solids obtained.
4. Find all sets of four real numbers x
1
, x
2
, x
3
, x
4
such that the sum of any one and
the product of the other three is equal to 2.
5. Consider OAB with acute angle AOB. Through a point M ,= O perpendiculars
are dawn to OA and OB, the feet of which are P and Q respectively. The point of
intersection of the altitudes of OPQ is H. What is the locus of H if M is permitted
to range over
(a) the side AB?
(b) the interior of OAB?
6. In a plane a set of n points (n 3) is given. Each pair of points is connected by a
segment. Let d be the length of the longest of these segment. We dene a diameter of
the set to be any connecting segment of length d. Prove that the number of diameters
of the given set is at most n.
1.8 8
th
IMO, Bulgaria, 1966
1. In a mathematical contest, three problems, A, B, C were posed. Among the par-
ticipants there were 25 students who solved at least one problem each. Of all the
1.9. 9
TH
IMO, YUGOSLAVIA, 1967 13
contestants who did not solve problem A, the number who solved B was twice the
number who solved C. The number of students who solved only problem A was one
more than the number of students who solved A and at least one other problem. How
many students solved only problem B?
2. Let a, b, c be the lengths of the sides of a triangle and , , , respectively, the
angles opposite these sides. Prove tat if a +b = tan

2
(a tan +b tan ), the triangle
is isosceles.
3. Prove: The sum of the distances of the vertices of a regular tetrahedron from the
centre of its circumscribed sphere is less than the sum of the distances of these vertices
from any other poin in space.
4. Prove that for every natural number n, and for every real number x ,=
k
2
t
(t any
non-negative integer and k any integer),
1
sin 2x
+
1
sin 4x
+ +
1
sin 2
n
x
= cot x cot 2
n
x
5. Solve the system of equations
[a
1
a
2
[x
2
+ [a
1
a
3
[x
3
+ [a
1
a
4
[x
4
= 1
[a
2
a
1
[x
2
+ [a
2
a
3
[x
3
+ [a
2
a
4
[x
4
= 1
[a
3
a
1
[x
1
+ [a
3
a
2
[x
2
+ [a
3
a
4
[x
4
= 1
[a
4
a
1
[x
1
+ [a
4
a
2
[x
2
+ [a
4
a
3
[x
3
= 1
where a
1
, a
2
, a
3
, a
4
are four dierent real numbers.
6. In the interior of sides BC, CA, AB of triangle ABC, any points K, L, M, respec-
tively, are selected. Prove that the area of at least one of the triangle AML, BKM, CLK
is less than or equal to one quarter of the area of ABC
1.9 9
th
IMO, Yugoslavia, 1967
1. Let ABCD be a parallelogram with side lengths AB = a, AD = 1, and with BAD =
. If ABD is acute, prove that the four circles of radius 1 with centers A, B, C, D
cover the parallelogram if and only if a cos +

3 sin .
2. Prove that if one and only one edge of a tetrahedron is greater than 1, then its volume
is smaller than or equal to
1
8
14 CHAPTER 1. INTERNATIONAL MATHEMATICS OLYMPIAD
3. Let k, m, n be natural numbers such that m +k + 1 is a prime greater than n + 1.
Let c
s
= s (s + 1). Prove that the product (c
m+1
c
k
) (c
m+2
c
k
) (c
m+n
c
k
) is
divisible by the product c
1
c
2
c
n
.
4. Let A
0
B
0
C
0
and A
1
B
1
C
1
be any two acute-angled triangles. Consider all triangles
ABC that are similar to A
1
B
1
C
1
and circumscribed about triangle A
0
B
0
C
0
(where A
0
lies on BC, B
0
on CA and C
0
on AB) Of all such triangles, determine the
one with maximum area, and construct it.
5. Consider the sequence c
n
, where
c
1
= a
1
+ a
2
+ + a
8
c
2
= a
2
1
+ a
2
2
+ + a
2
8
.
.
.
c
n
= a
n
1
+ a
n
2
+ + a
n
8
.
.
.
in which a
1
, a
2
, . . . , a
8
are real numbers not all equal to zero. Suppose that an innite
number of terms of the sequence c
n
are equal to zero. Find all natural numbers for
which c
n
= 0.
6. In a sport contest, there were m medals awarded on n successive days (n > 1). On
the rst day, one medal and
1
7
of the remaining medals were awarded. On the second
day, two medals and
1
7
of the now remaining medals were awarded; and so on. On
the n-th and last day, the remaining n medals were awarded. How many days did
the contest last. and how many medals were awarded altogether?
1.10 10
th
IMO, USSR, 1968
1. Prove that there is one and only one triangle whose side lengths are consecutive
integers, and one of whose angles is twice as large as another.
2. Find all natural numbers x such that the product of their digits (in decimal notation)
is equal to x
2
10x 22.
1.10. 10
TH
IMO, USSR, 1968 15
3. Consider the system of equations:
ax
2
1
+ bx
1
+ c = x
2
ax
2
2
+ bx
2
+ c = x
3
.
.
.
ax
2
n1
+ bx
n1
+ c = x
n
ax
2
n
+ bx
n
+ c = x
1
with unknowns x
1
, x
2
, . . . , x
n
, where a, b, c are real and a ,= 0. Let = (b 1)
2

4ac. Prove that for this system


(a) If < 0, ther is no solution,
(b) If = 0, ther is exactly one solution,
(c) If > 0, ther is more than one solution.
4. Prove than in every tetrahedon there is a vertex such that the three edges meeting
there have lengths which are the sides of a triangle.
5. Let f be a real-valued function dened for all real numbers x such that, for some
positive constant a, the equation
f (x +a) =
1
2
+
_
f (x) [f (x)]
2
holds for all x
(a) Prove that the function f is periodic (i.e. there exists a positive number b such
that f (x +b) = f (x) for all x)
(b) For a = 1, give an example of a non-constant function with the requiered prop-
erties.
6. For every natural number n, evaluate the sum

k=0
_
n + 2
k
2
k+1
_
=
_
n + 1
2
_
+
_
n + 2
4
_
+ +
_
n + 2
k
2
k+1
_
+
(the symbol x| denotes the greatest integer not exceding x).
16 CHAPTER 1. INTERNATIONAL MATHEMATICS OLYMPIAD
1.11 11
th
IMO, Romania, 1969
1. Prove that there are innitely many numbers a with the following property: the
number z = n
4
+a is not prime for any natural number n.
2. Let a
1
, a
2
, . . . , a
n
be real variable, and
f (x) = cos (a
1
+x) +
1
2
cos (a
2
+x) +
1
4
cos (a
3
+x) + +
1
2
n1
cos (a
n
+x)
Given that f (x
1
) = f (x
2
) = 0, prove that x
2
x
1
= m for some integer m.
3. For each value of k = 1, 2, 3, 4, 5, nd necessary and sucient conditions on the
number a > 0 so that there exist a tetrahedron with k edges of length a, and the
remaining 6 k edges of lenght 1.
4. A semicircular arc is drawn on AB as diameter. C is a point on other than A and
B, and D is the foot of the perpendicular from C to AB. We consider three circles

1
,
2
,
3
, all tangent to the line AB. Of these,
1
is inscrived in ABC, while
2
and
3
are both tangent to CD and to , one on each side of CD. Prove that
1
,
2
and
3
have a second tangent in common.
5. Given n > 4 points in the plane such that no three are collinear. Prove that there
are at least
_
n3
2
_
convex quadrilaterals whose vertices are four of the given points.
6. Prove that for all real numbers x
1
, x
2
, y
1
, y
2
, z
1
, z
2
with x
1
> 0, x
2
> 0, x
1
y
1
z
2
1
>
0, x
2
y
2
z
2
2
> 0, the inequality
8
(x
1
+x
2
) (y
1
+y
2
) (z
1
+z
2
)
2

1
x
1
y
1
z
2
1
+
1
x
2
y
2
z
2
2
is satised. Give necessary and sucient conditions for equality.
1.12 12
th
IMO, Hungary, 1970
1. Let M be a point on the sede AB of ABC. Let r
1
, r
2
and r be the radii of the
inscribed circles of the triangles AMC, BMC and ABC. Let q
1
, q
2
and q be
the radii of the excribed circles of the same triangles that lie in the angle ACB.
Prove that
r
1
q
1

r
2
q
2
=
r
q
1.12. 12
TH
IMO, HUNGARY, 1970 17
2. Let a, b and n be integers greater than 1, and let a and b be the two bases of two
number systems. A
n1
and A
n
are numbers in the system with base a and B
n1
and
B
n
are numbers in the system with base b; these are related as follows:
A
n
= x
n
x
n1
x
0
A
n1
= x
n1
x
n2
x
0
B
n
= x
n
x
n1
x
0
B
n1
= x
n1
x
n2
x
0
such that x
n
,= 0 and x
n1
,= 0
1
. Prove that
A
n1
A
n
<
B
n1
B
n
a > b
3. The real numbers a
0
, a
1
, . . . , a
n
, . . . satisfy the condition 1 = a
0
a
1
a
2

a
n
. The numbers b
1
, b
2
, . . . , b
n
, . . . are dened by
b
n
=
n

k=1
_
1
a
k1
a
k
_
1

a
k
(a) Prove that 0 b
n
< 2 for all n.
(a) Given c with 0 c < 2, prove that there exist numbers a
0
, a
1
, . . . such that
b
n
> c for large enough n.
4. Find the set of all positive integers n with the property that the set n, n + 1, n +
2, n + 3, n + 4, n + 5 can be partitioned into sets such that the product of the
numbers in one set equals the product of the numbers in the other set
5. In the tetrahedron ABCD, the angle BDC is a right angle. Suppose that the foot
H of the perpendicular from D to the plane ABC is the intersection of the altitudes
of ABC. Prove that
(AB +BC +CA)
2
6
_
AD
2
+BD
2
+CD
2
_
For what tetrahedra does equality hold?
6. In the plane are 100 points, no three of them are collinear. Consider all posible
triangles having these points as vertices. Prove that no more than 70% of these
triangles are acute-angled.
1
The x
i
s are the digits in the respective bases, and of course, all of them are lower than the lowest base
18 CHAPTER 1. INTERNATIONAL MATHEMATICS OLYMPIAD
1.13 13
th
IMO, Czechoslovakia, 1971
1. Prove that the following assertion is true for n = 3 and n = 5, and that it is false
for every other natural number n > 2. If a
1
, a
2
, . . . , a
n
are arbitrary real numbers,
then:
(a
1
a
2
) (a
1
a
3
) (a
1
a
n
) + (a
2
a
1
) (a
2
a
3
) (a
2
a
n
)
+ + (a
n
a
1
) (a
n
a
2
) (a
n
a
n1
) 0
2. Consider a convex polyhedron P
1
with nine vertices A
1
A
2
, ..., A
9
; let P
i
be the poly-
hedron obtained from P
1
by a translation that moves vertex A
1
to A
i
(i = 2, 3, ..., 9).
Provethat at least two of the polyhedra P
1
, P
2
, ..., P
9
have an interiorpoint in common.
3. Prove that the set of integers of the form 2
k
3(k = 2, 3, ...) contains an innite
subset in which every two members are relatively prime.
4. All the faces of tetrahedron ABCD are acute-angled triangles. We consider all closed
polygonal paths of the form XY ZTX dened as follows: X is a point on edge AB
distinct from A and B; similarly, Y, Z, T are interior points of edges BCCD, DA,
respectively. Prove:
(a) If DAB +BCD ,= CDA+ABC, then among thepolygonal paths, there is
none of minimal length.
(b) If DAB + BCD = CDA + ABC, then there areinnitely many shortest
polygonal paths, their common length being 2AC sin(/2), where = BAC +
CAD +DAB.
5. Prove that for every natural number m, there exists a nite set S of points in a plane
with the following property: For every point A in S, there are exactly m points in S
which are at unit distance from A.
6. Let A = (a
ij
)(i, j = 1, 2, ..., n) be a square matrix whose elements are non-negative
integers. Suppose that whenever an element a
ij
= 0, the sum of the elements in the
ith row and the jth column is n. Prove that the sum of all the elements of the
matrix is n
2
/2.
1.14 14
th
IMO, USSR, 1972
1. Prove that from a set of ten distinct two-digit numbers (in the decimalsystem), it is
possible to select two disjoint subsets whose members havethe same sum.
1.15. 15
TH
IMO, USSR, 1973 19
2. Prove that if n 4, every quadrilateral that can be inscribed in acircle can be
dissected into n quadrilaterals each of which is inscribablein a circle.
3. Let m and n be arbitrary non-negative integers. Prove that
(2m)!(2n)!
m!n!(m +n)!
is an integer. (0! = 1)
4. Find all solutions (x
1
, x
2
, x
3
, x
4
, x
5
) of the system of inequalities
(x
2
1
x
3
x
5
)(x
2
2
x
3
x
5
) 0
(x
2
2
x
4
x
1
)(x
2
3
x
4
x
1
) 0
(x
2
3
x
5
x
2
)(x
2
4
x
5
x
2
) 0
(x
2
4
x
1
x
3
)(x
2
5
x
1
x
3
) 0
(x
2
5
x
2
x
4
)(x
2
1
x
2
x
4
) 0
where x
1
, x
2
, x
3
, x
4
, x
5
are positive real numbers.
5. Let f and g be real-valued functions dened for all real values of xand y, and satisfying
the equation
f(x +y) +f(x y) = 2f(x)g(y)
for all x, y. Prove that if f(x) is not identically zero, and if [f(x)[ 1 for all x, then
[g(y)[ 1 for all y.
6. Given four distinct parallel planes, prove that there exists a regular tetrahedron with
a vertex on each plane.
1.15 15
th
IMO, USSR, 1973
1. Point O lies on line g;

OP
1
,

OP
2
, . . . ,

OP
n
are unit vectors such that points P
1
, P
2
, ..., P
n
all lie in a plane containing g and on one side of g. Prove that if n is odd,

OP
1
+

OP
2
+ +

OP
n

1
Here

OM

denotes the length of vector



OM.
20 CHAPTER 1. INTERNATIONAL MATHEMATICS OLYMPIAD
2. Determine whether or not there exists a nite set M of points in spacenot lying in
the same plane such that, for any two points A and B of M,one can select two other
points C and D of M so that lines AB and CD are parallel and not coincident.
3. Let a and b be real numbers for which the equation
x
4
+ax
3
+bx
2
+ax + 1 = 0
has at least one real solution. For all such pairs (a, b), nd the minimum value of
a
2
+b
2
.
4. A soldier needs to check on the presence of mines in a region having theshape of an
equilateral triangle. The radius of action of his detector isequal to half the altitude of
the triangle. The soldier leaves from one vertex of the triangle. What path should he
follow in order to travel the least possible distance and still accomplish his mission?
5. G is a set of non-constant functions of the real variable x of the form f(x) = ax+b, a
and b are real numbers, and G has the following properties:
(a) If f and g are in G, then g f is in G; here (g f)(x) = g[f(x)].
(b) If f is in G, then its inverse f
1
is in G; here the inverse of f(x) = ax + b is
f
1
(x) = (x b)/a.
(c) For every f in G, there exists a real number x
f
such that f(x
f
) = x
f
.
Prove that there exists a real number k such that f(k) = k for all f in G.
6. Let a
1
, a
2
, ..., a
n
be n positive numbers, and let q be a givenreal number such that
0 < q < 1. Find n numbers b
1
, b
2
, ..., b
n
forwhich
(a) a
k
< b
k
for k = 1, 2, , n,
(b) q <
b
k+1
b
k
<
1
q
for k = 1, 2, ..., n 1,
(c) b
1
+b
2
+ +b
n
<
1+q
1q
(a
1
+ a
2
+ +a
n
).
1.16 16
th
IMO, West Germany, 1974
1. Three players A, B and C play the following game: On each of three cardsan integer
is written. These three numbers p, q, r satisfy 0 < p < q < r. Thethree cards are
shued and one is dealt to each player. Each then receivesthe number of counters
indicated by the card he holds. Then the cards areshued again; the counters remain
with the players.
1.17. 17
TH
IMO, BULGARIA, 1975 21
This process (shuing, dealing, giving out counters) takes place for at least two
rounds. After the last round, A has 20 counters in all, B has 10 and C has 9. At the
last round B received r counters. Who received q counters on the rst round?
2. In the triangle ABC prove that there is a point D on side AB suchthat CD is the
geometric mean of AD and DB if and only if
sin Asin B sin
2
C
2
.
3. Prove that the number

n
k=0
_
2n+1
2k+1
_
2
3k
is not divisible by 5 for any integer n 0.
4. Consider decompositions of an 8 8 chessboard into p non-overlapping rectangles
subject to the following conditions:
(i) Each rectangle has as many white squares as black squares.
(ii) If a
i
is the number of white squares in the i-th rectangle, then a
1
< a
2
< < a
p
.
Find the maximum value of p for which such a decomposition is possible. For this
value of p, determine all possible sequences a
1
, a
2
, , a
p
.
5. Determine all possible values of
S =
a
a +b +d
+
b
a +b +c
+
c
b +c +d
+
d
a +c +d
where a, b, c, d are arbitrary positive numbers.
6. Let P be a non-constant polynomial with integer coecients. If n(P) isthe number
of distinct integers k such that (P(k))
2
= 1, prove that n(P) deg(P) 2, where
deg(P) denotes the degree of the polynomial P.
1.17 17
th
IMO, Bulgaria, 1975
1. Let x
i
, y
i
(i = 1, 2, ..., n) be real numbers such that
x
1
x
2
x
n
and y
1
y
2
y
n
Prove that, if z
1
, z
2
, , z
n
is any permutation of y
1
, y
2
, , y
n
, then
n

i=1
(x
i
y
i
)
2

i=1
(x
i
z
i
)
2
22 CHAPTER 1. INTERNATIONAL MATHEMATICS OLYMPIAD
2. Let a
1
, a
2
, a
3,
be an innite increasing sequence of positive integers. Prove that for
every p 1 there are innitely many a
m
which can be written in the form
a
m
= xa
p
+ya
q
with x, y positive integers and q > p.
3. On the sides of an arbitrary triangle ABC, triangles ABR, BCP, CAQ areconstructed
externally with CBP = CAQ = 45

, BCP = ACQ = 30

, ABR = BAR =
15

. Prove that QRP = 90

and QR = RP.
4. When 4444
4444
is written in decimal notation, the sum of its digits is A. Let B be
the sum of the digits of A. Find the sum of the digits of B. (A and B are written in
decimal notation.)
5. Determine, with proof, whether or not one can nd 1975 points on the circumference
of a circle with unit radius such that the distance between any two of them is a
rational number.
6. Find all polynomials P, in two variables, with the following properties:
(i) for a positive integer n and all real t, x, y
P(tx, ty) = t
n
P(x, y)
(that is, P is homogeneous of degree n),
(ii) for all real a, b, c,
P(b +c, a) +P(c +a, b) +P(a +b, c) = 0
(iii) P(1, 0) = 1.
1.18 18
th
IMO, Austria, 1976
1. In a plane convex quadrilateral of area 32, the sum of the lengths of two opposite
sides and one diagonal is 16. Determine all possible lengths ofthe other diagonal.
2. Let P
1
(x) = x
2
2 and P
j
(x) = P
1
(P
j1
(x)) for j = 2, 3, .Show that, for any
positive integer n, the roots of the equation P
n
(x) = x are real and distinct.
1.19. 19
TH
IMO, YUGOSLAVIA, 1977 23
3. A rectangular box can be lled completely with unit cubes. If one places as many
cubes as possible, each with volume 2, in the box, so that their edges are parallel
to the edges of the box, one can ll exactly 40% ofthe box. Determine the possible
dimensions of all such boxes.
4. Determine, with proof, the largest number which is the product of positiveintegers
whose sum is 1976.
5. Consider the system of p equations in q = 2p unknowns x
1
, x
2
, , x
q
:
a
11
x
1
+a
12
x
2
+ +a
1q
x
q
= 0
a
21
x
1
+a
22
x
2
+ +a
2q
x
q
= 0

a
p1
x
1
+a
p2
x
2
+ +a
pq
x
q
= 0
with every coecient a
ij
member of the set 1, 0, 1. Prove that the system has a
solution (x
1
, x
2
, , x
q
) such that
(a) all x
j
(j = 1, 2, ..., q) are integers,
(b) there is at least one value of j for which x
j
,= 0,
(c) [x
j
[ q(j = 1, 2, ..., q).
6. A sequence u
n
is dened by
u
0
= 2, u
1
= 5/2, u
n+1
= u
n
(u
2
n1
2) u
1
for n = 1, 2,
Prove that for positive integers n,
[u
n
] = 2
[2
n
(1)
n
]/3
where [x] denotes the greatest integer x.
1.19 19
th
IMO, Yugoslavia, 1977
1. Equilateral triangles ABK, BCL, CDM, DAN are constructed inside the
square ABCD. Prove that the midpoints of the four segments KL, LM, MN, NK
and the midpoints of the eight segments AK, BK, BL, CL, CM, DM, DN, AN
are the twelve vertices of a regular dodecagon.
24 CHAPTER 1. INTERNATIONAL MATHEMATICS OLYMPIAD
2. In a nite sequence of real numbers the sum of any seven successive terms is nega-
tive, and the sum of any eleven successive terms is positive.Determine the maximum
number of terms in the sequence.
3. Let n be a given integer > 2, and let V
n
be the set of integers 1+kn, where k = 1, 2, ....
A number m V
n
is called indecomposable in V
n
if there do not exist numbers
p, q V
n
such that pq = m. Prove that there exists a number r V
n
that can be
expressed as the product of elements indecomposable in V
n
in more than one way.
(Products which dier only in the order of their factors will be considered the same.)
4. Four real constants a, b, A, B are given, and
f() = 1 a cos b sin Acos 2 Bsin 2
Prove that if f() 0 for all real , then
a
2
+b
2
2 and A
2
+B
2
1.
5. Let a and b be positive integers. When a
2
+ b
2
is divided by a + b,the quotient is q
and the remainder is r. Find all pairs (a, b) suchthat q
2
+r = 1977.
6. Let f(n) be a function dened on the set of all positive integers and having all its
values in the same set. Prove that if f(n + 1) > f(f(n)) for each positive integer n,
then f(n) = n for each n.
1.20 20
th
IMO, Romania, 1978
1. m and n are natural numbers with 1 m < n. In their decimal representations, the
last three digits of 1978
m
are equal, respectively, to the last three digits of 1978
n
.
Find m and n such that m +n has its least value.
2. P is a given point inside a given sphere. Three mutually perpendicular rays from P
intersect the sphere at points U, V, and W; Q denotes the vertex diagonally opposite
to P in the parallelepiped determined by PU, PV, and PW. Find the locus of Q for
all such triads of rays from P.
3. The set of all positive integers is the union of two disjoint subsets f(1), f(2), ..., f(n), ...,
g(1), g(2), ..., g(n), ..., where
f(1) < f(2) < < f(n) <
1.21. 21
ST
IMO, UNITED KINGDOM, 1979 25
g(1) < g(2) < < g(n) <
and
g(n) = f(f(n)) + 1 for all n 1
Determine f(240).
4. In triangle ABC, AB = AC. A circle is tangent internally to thecircumcircle of
triangle ABC and also to sides AB, AC at P, Q, respectively. Prove that the
midpoint of segment PQ is the center of the incircle of triangle ABC.
5. Let a
k
(k = 1, 2, 3, ..., n, ...) be a sequence of distinct positive integers. Prove that
for all natural numbers n,
n

k=1
a
k
k
2

n

k=1
1
k
6. An international society has its members from six dierent countries. The list of
members contains 1978 names, numbered 1, 2, ..., 1978. Prove that there is at least
one member whose number is the sum of thenumbers of two members from his own
country, or twice as large as the numberof one member from his own country.
1.21 21
st
IMO, United Kingdom, 1979
1. Let p and q be natural numbers such that
p
q
= 1
1
2
+
1
3

1
4
+
1
1318
+
1
1319
Prove that p is divisible by 1979.
2. A prism with pentagons A
1
A
2
A
3
A
4
A
5
and B
1
B
2
B
3
B
4
B
5
as top and bottom faces is
given. Each side of the two pentagons and each of the line-segments A
i
B
j
for all
i, j = 1, ..., 5, is colored either red or green. Every triangle whose vertices are vertices
of the prism and whose sides have all been colored has two sides of a dierent color.
Show that all 10 sides of the top and bottom faces are the same color.
3. Two circles in a plane intersect. Let A be one of the points of intersection. Starting
simultaneously from A two points move with constant speeds, each point travelling
along its own circle in the same sense. The two points return to A simultaneously
after one revolution. Prove that there is a xed point P in the plane such that, at
any time, the distances from P to the moving points are equal.
26 CHAPTER 1. INTERNATIONAL MATHEMATICS OLYMPIAD
4. Given a plane , a point P in this plane and a point Q not in , nd all points R in
such that the ratio (QP +PA)/QR is a maximum.
5. Find all real numbers a for which there exist non-negative real numbers x
1
, x
2
, x
3
, x
4
, x
5
satisfying the relations
5

k=1
kx
k
= a,
5

k=1
k
3
x
k
= a
2
,
5

k=1
k
5
x
k
= a
3
6. Let A and E be opposite vertices of a regular octagon. A frog starts jumping at
vertex A. From any vertex of the octagon except E, it may jump to either of the two
adjacent vertices. When it reaches vertex E, the frog stops and stays there.. Let a
n
be the number of distinct paths of exactly n jumps ending at E. Prove that a
2n1
= 0,
a
2n
=
1

2
(x
n1
y
n1
), n = 1, 2, 3, ,
where x = 2 +

2 and y = 2

2
Note. A path of n jumps is a sequence of vertices (P
0
, ..., P
n
) such that
(i) P
0
= A, P
n
= E;
(ii) for every i, 0 i n 1, P
i
is distinct from E;
(iii) for every i, 0 i n 1, P
i
and P
i+1
are adjacent.
1.22 22
nd
IMO, Washington, USA, 1981
1. P is a point inside a given triangle ABC.D, E, F are the feet of the perpendiculars
from P to the lines BC, CA, AB respectively. Find all P for which
BC
PD
+
CA
PE
+
AB
PF
is least.
2. Let 1 r n and consider all subsets of r elements of theset 1, 2, ..., n. Each
of these subsets has a smallest member. Let F(n, r) denote the arithmetic mean of
these smallest numbers; prove that
F(n, r) =
n + 1
r + 1
1.23. 23
RD
IMO, BUDAPEST, HUNGARY, 1982 27
3. Determine the maximum value of m
3
+ n
3
,where m and n are integers satisfying
m, n 1, 2, ..., 1981 and (n
2
mn m
2
)
2
= 1
4. (a) For which values of n > 2 is there a set of n consecutive positive integers such
that the largest number in the set is a divisor of the least common multiple of the
remaining n 1 numbers?
(b) For which values of n > 2 is there exactly one set having the stated property?
5. Three congruent circles have a common point O and lie inside a given triangle. Each
circle touches a pair of sides of the triangle. Prove that the incenter and the circum-
center of the triangle and the point O are collinear.
6. The function f(x, y) satises
(1) f(0, y) = y + 1,
(2)f(x + 1, 0) = f(x, 1),
(3) f(x + 1, y + 1) = f(x, f(x + 1, y)),
for all non-negative integers x, y. Determine f(4, 1981).
1.23 23
rd
IMO, Budapest, Hungary, 1982
1. The function f(n) is dened for all positive integers n and takes on non-negative
integer values. Also, for all m, n
f(m +n) f(m) f(n) = 0 or 1
f(2) = 0, f(3) > 0, and f(9999) = 3333
Determine f(1982).
2. A non-isosceles triangle A
1
A
2
A
3
is given with sides a
1
, a
2
, a
3
(a
i
is the side opposite
A
i
). For all i = 1, 2, 3, M
i
is the midpoint of side a
i
, and T
i
. is the pointwhere the
incircle touches side a
i
. Denote by S
i
the reection of T
i
in the interior bisector of
angle A
i
. Prove that the lines M
1
, S
1
, M
2
S
2
, and M
3
S
3
are concurrent.
3. Consider the innite sequences x
n
of positive real numbers with the following prop-
erties:
x
0
= 1, and for all i 0, x
i+1
x
i
28 CHAPTER 1. INTERNATIONAL MATHEMATICS OLYMPIAD
(a) Prove that for every such sequence, there is an n 1 such that
x
2
0
x
1
+
x
2
1
x
2
+ +
x
2
n1
x
n
3.999
(b) Find such a sequence for which
x
2
0
x
1
+
x
2
1
x
2
+ +
x
2
n1
x
n
< 4
4. Prove that if n is a positive integer such that the equation x
3
3xy
2
+ y
3
= n has
a solution in integers (x, y), then it has at least three suchsolutions. Show that the
equation has no solutions in integers when n = 2891
5. The diagonals AC and CE of the regular hexagon ABCDEF are divided by the
inner points M and N, respectively, so that
AM
AC
=
CN
CE
= r
Determine r if B, M, and N are collinear.
6. Let S be a square with sides of length 100, and let L be a path within S which does
not meet itself and which is composed of line segments A
0
A
1
, A
1
A
2
, , A
n1
A
n
with
A
0
,= A
n
. Suppose that for every point P of the boundary of S there is a point of L
at a distance from P not greater than 1/2. Prove that there are two points X and Y
in L such that the distance between X and Y is not greater than 1, and the length
of that part of L which lies between X and Y is not smaller than 198.
1.24 24
th
IMO, Paris, France, 1983
1. Find all functions f dened on the set of positive real numbers which take positive
real values and satisfy the conditions:
(i) f(xf(y)) = yf(x) for all positive x, y;
(ii) f(x) 0 as x
2. Let A be one of the two distinct points of intersection of two unequal coplanar circles
C
1
and C
2
with centers O
1
and O
2
, respectively. One of the common tangents to the
circles touches C
1
at P
1
and C
2
at P
2
, while the other touches C
1
at Q
1
and C
2
at
Q
2
. Let M
1
be the midpoint of P
1
Q
1
,and M
2
be the midpoint of P
2
Q
2
. Prove that
O
1
AO
2
= M
1
AM
2
.
1.25. 25
TH
IMO, PRAGUE, CZECHOSLOVAKIA, 1984 29
3. Let a, b and c be positive integers, no two of which have a common divisor greater
than 1. Show that 2abc ab bc ca is the largest integer which cannot be expressed
in the form xbc +yca +zab,where x, y and z are non-negative integers.
4. Let ABC be an equilateral triangle and c the set of all points contained in the three
segments AB, BC and CA (including A, B and C). Determine whether, for every
partition of c into two disjoint subsets, at least one of the two subsets contains the
vertices of a right-angled triangle. Justify your answer.
5. Is it possible to choose 1983 distinct positive integers, all less than or equal to 10
5
,
no three of which are consecutive terms of an arithmetic progression? Justify your
answer.
6. Let a, b and c be the lengths of the sides of a triangle. Prove that
a
2
b(a b) +b
2
c(b c) +c
2
a(c a) 0
Determine when equality occurs.
1.25 25
th
IMO, Prague, Czechoslovakia, 1984
1. Prove that
0 yz +zx +xy 2xyz
7
27
where x, y and z arenon-negative real numbers for which x +y +z = 1.
2. Find one pair of positive integers a and b such that:
(i) ab(a +b) is not divisible by 7;
(ii) (a +b)
7
a
7
b
7
is divisible by 7
7
.
Justify your answer.
3. In the plane two dierent points O and A are given. For each point X of the plane,
other than O, denote by a(X) the measure of the angle between OA and OX in
radians, counterclockwise from OA(0 a(X) < 2). Let C(X) be the circle with
center O and radius of length OX +a(X)/OX. Each point of the plane is colored by
one of a nite number ofcolors. Prove that there exists a point Y for which a(Y ) > 0
such that its color appears on the circumference of the circle C(Y ).
30 CHAPTER 1. INTERNATIONAL MATHEMATICS OLYMPIAD
4. Let ABCD be a convex quadrilateral such that the line CD is a tangent to the circle
on AB as diameter. Prove that the line AB is a tangent to the circle on CD as
diameter if and only if the lines BC and AD are parallel.
5. Let d be the sum of the lengths of all the diagonals of a plane convex polygon with
n vertices (n > 3), and let p be its perimeter. Prove that
n 3 <
2d
p
<
_
n
2
_ _
n + 1
2
_
2
where [x] denotes the greatest integer not exceeding x.
6. Let a, b, c and d be odd integers such that 0 < a < b < c < d and ad = bc. Prove that
if a +d = 2
k
and b +c = 2
m
for some integers k and m, then a = 1.
1.26 26
th
IMO, Helsinki, Finland, 1985
1. A circle has center on the side AB of the cyclic quadrilateral ABCD. The other three
sides are tangent to the circle. Prove that AD +BC = AB
2. Let n and k be given relatively prime natural numbers, k < n. Each number in the
set M = 1, 2, ..., n 1 is colored either blue or white. It is given that
(i) for each i M, both i and n i have the same color;
(ii) for each i M, i ,= k both i and [i k[ have the same color. Prove that all
numbers in M must have the same color.
3. For any polynomial P(x) = a
0
+a
1
x+ +a
k
x
k
with integer coecients, the number
of coecients which are odd is denoted by w(P). For i = 0, 1, ..., let Q
i
(x) = (1 +x)
i
.
Prove that if i
1
i
2
, ..., i
n
are integers such that 0 i
1
< i
2
< < i
n
then
w(Q
i
1
+Q
i
2
+ +Q
in
) w(Q
i
1
)
4. Given a set M of 1985 distinct positive integers, none of which has a prime divisor
greater than 26. Prove that M contains at least one subset of four distinct elements
whose product is the fourth power of an integer.
5. A circle with center O passes through the vertices A and C of triangle ABC and
intersects the segments AB and BC again at distinct points K and N, respectively.
The circumscribed circles of the triangles ABC and EBN intersect at exactly
two distinct points B and M. Provethat angle OMB is a right angle.
1.27. 27
TH
IMO, WARSAW, POLAND, 1986 31
6. For every real number x
1
, construct the sequence x
1
, x
2
, ... by setting
x
n+1
= x
n
_
x
n
+
1
n
_
for each n 1
Prove that there exists exactly one value of x
1
for which
0 < x
n
< x
n+1
< 1
for every n.
1.27 27
th
IMO, Warsaw, Poland, 1986
1. Let d be any positive integer not equal to 2, 5, or 13. Show that one canind distinct
a, b in the set 2, 5, 13, d such that ab 1 is not perfect square.
2. A triangle A
1
A
2
A
3
and a point P
0
are given in the plane. We dene A
s
= A
s3
for
all s 4. We construct a set of points P
1
, P
2
, P
3
, . . . , such that P
k+1
is the image
of P
k
under a rotation withenter A
k+1
through angle 120

clockwise (for k = 0, 1,
2,ldots). Prove that if P
1986
= P
0
, then the triangle A
1
A
2
A
3
equilateral.
3. To each vertex of a regular pentagon an integer is assigned in such a way that the sum
of all ve numbers is positive. If three consecutive vertices are assigned the numbers
x, y, z respectively and y < 0 then the following operation is allowed: the numbers
x, y, z are replaced by x +y, y, z +y respectively. Such an operation is performed
repeatedly as long as at least one of the ve numbers is negative. Determine whether
this procedure necessarily comes to and end after a nite number of steps.
4. Let A, B be adjacent vertices of a regular n-gon (n 5) in thelane having center at
O. A triangle XY Z, which is congruent to andnitially conincides with OAB, moves
in the plane in such a way that Y and Z each trace out the whole boundary of the
polygon, X remaining inside the polygon. Find the locus of X.
5. Find all functions f, dened on the non-negative real numbers and taking non-
negative real values, such that:
(i) f(xf(y))f(y) = f(x +y) for all x, y 0,
(ii) f(2) = 0,
(iii) f(x) ,= 0 for 0 x < 2.
32 CHAPTER 1. INTERNATIONAL MATHEMATICS OLYMPIAD
6. One is given a nite set of points in the plane, each point having integeroordinates.
Is it always possible to color some of the points in the set rednd the remaining points
white in such a way that for any straight line Larallel to either one of the coordinate
axes the dierence (in absolutealue) between the numbers of white point and red
points on L is not greaterhan 1?
1.28 28
th
IMO, Havana, Cuba , 1987
1. Let p
n
(k) be the number of permutations of the set 1, . . . , n, n 1, which have
exactly k xed points. Prove that
n

k=0
k p
n
(k) = n!
(Remark: A permutation f of a set S is a one-to-one mapping of S onto itself. An
element i in S is called a xed point of the permutation f if f(i) = i.)
2. In an acute-angled triangle ABC the interior bisector of the angle A intersects
BC at L and intersects the circumcircle of ABC again at N. From point L per-
pendiculars are drawn to AB and AC, the feet of theseerpendiculars being K and M
respectively. Prove that the quadrilateral AKNM and the triangle ABC have equal
areas.
3. Let x
1
, x
2
, . . . , x
n
be real numbers satisfying x
2
1
+x
2
2
+ + x
2
n
= 1. Prove that for
every integer k 2 there are integers a
1
, a
2
, . . . , a
n
, not all 0, such that [a
i
[ k 1
For all i and
[a
1
x
1
+a
1
x
2
+ +a
n
x
n
[
(k 1)

n
k
n

4. Prove that there is no function f from the set of non-negative integers into itself such
that f(f(n)) = n + 1987 for every n.
5. Let n be an integer greater than or equal to 3. Prove that there is a set of n points
in the plane such that the distance between any two points is irrational and each set
of three points determines a non-degenerate triangle with rational area.
6. Let n be an integer greater than or equal to 2. Prove that if k
2
+ k +n is prime for
all integers k such that 0 k
_
n
3
, then k
2
+k +n is prime for all integers k such
that 0 k n 2.
1.29. 29
TH
IMO, CAMBERRA, AUSTRALIA, 1988 33
1.29 29
th
IMO, Camberra, Australia, 1988
1. Consider two coplanar circles of radii R and r (R > r) with the same center. Let P
be a xed point on the smaller circle and B a variable point on the larger circle. The
line BP meets the larger circle again at C. The perpendicular l to BP at P meets
the smaller circle again at A. (If l is tangent to the circle at P then A = P.)
(i) Find the set of values of BC
2
+CA
2
+AB
2
.
(ii) Find the locus of the midpoint of BC.
2. Let n be a positive integer and let A
1
, A
2
, . . . , A
2n+1
be subsets of a set B. Suppose
that
(a) Each A
i
has exactly 2n elements,
(b) Each A
i
A
j
(1 i < j 2n + 1) contains exactly one element, and
(c) Every element of B belongs to at least two of the A
i
.
For which values of n can one assign to every element of B one of the numbers 0 and
1 in such a way that A
i
has 0 assigned to exactly n of its elements?
3. A function f is dened on the positive integers by
f(1) = 1, f(3) = 3,
f(2n) = f(n),
f(4n + 1) = 2f(2n + 1) f(n),
f(4n + 3) = 3f(2n + 1) 2f(n),
for all positive integers n.
Determine the number of positive integers n, less than or equal to 1988, for which
f(n) = n.
4. Show that set of real numbers x which satisfy the inequality
70

k=1
k
x k

5
4
is a union of disjoint intervals, the sum of whose lengths is 1988.
5. ABC is a triangle right-angled at A, and D is the foot of the altituderom A. The
straight line joining the incenters of the triangles ABD, ACD intersects the sides
AB, AC at the points K, L respectively. S and T denote the areas of the triangles
ABC and AKL respectively.how that S 2T.
34 CHAPTER 1. INTERNATIONAL MATHEMATICS OLYMPIAD
6. Let a and b be positive integers such that ab + 1 divides a
2
+b
2
. Show that
a
2
+b
2
ab + 1
is the square of an integer.
1.30 30
th
IMO, Braunschweig, West Germany, 1989
1. Prove that the set 1, 2, . . . , 1989 can be expressed as the disjoint union of subsets
A
i
(i = 1, 2, . . . , 117) such that:
(i) Each A
i
contains 17 elements;
(ii) The sum of all the elements in each A
i
is the same.
2. In an acute-angled triangle ABC the internal bisector of angle A meets their cumcircle
of the triangle again at A
1
. Points B
1
and C
1
are dened similarly. Let A
0
be the
point of intersection of the line AA
1
with the external bisectors of angles B and C.
Points B
0
and C
0
are dened similarly. Prove that:
(i) The area of the triangle A
0
B
0
C
0
is twice the area of the hexagon AC
1
BA
1
CB
1
.
(ii) The area of the triangle A
0
B
0
C
0
is at least four times the area of the triangle
ABC.
3. Let n and k be positive integers and let S be a set of n points in the plane such that
(i) No three points of S are collinear, and
(ii) For any point P of S there are at least k points of S equidistant from P.
Prove that:
k <
1
2
+

2n
4. Let ABCD be a convex quadrilateral such that the sides AB, AD, BC satisfy AB =
AD + BC. There exists a point P inside the quadrilateral at a distance h from the
line CD such that AP = h +AD and BP = h +BC. Show that:
1

AD
+
1

BC
5. Prove that for each positive integer n there exist n consecutive positive integers none
of which is an integral power of a prime number.
1.30. 30
TH
IMO, BRAUNSCHWEIG, WEST GERMANY, 1989 35
6. A permutation (x
1
, x
2
, . . . , x
m
) of the set 1, 2, . . . , 2n, where n is a positive integer,
is said to have property P if [x
i
x
i+1
[ = n for at least one i in 1, 2, . . . , 2n 1.
Show that, for each n, there are more permutations with property P than without.
36 CHAPTER 1. INTERNATIONAL MATHEMATICS OLYMPIAD
1.31 31
st
IMO, Beijing, Peoples Republic of China,
1990
1. Chords AB and CD of a circle intersect at a point E inside the circle. Let M be an
interior point of the segment EB. The tangent line at E to the circle through D, E,
and M intersects the lines BC and AC at F and G, respectively. If
AM
AB
= t,
nd
EG
EF
in terms of t.
2. Let n 3 and consider a set E of 2n 1 distinct points on a circle. Suppose that
exactly k of these points are to be colored black. Such a coloring is good if there
is at least one pair of black points such that the interior of one of the arcs between
them contains exactly n points from E. Find the smallest value of k so that every
such coloring of k points of E is good.
3. Determine all integers n > 1 such that
2
n
+ 1
n
2
is an integer.
4. Let Q
+
be the set of positive rational numbers. Construct a function f : Q
+
Q
+
such that
f(xf(y)) =
f(x)
y
for all x, y in Q
+
.
5. Given an initial integer n
0
> 1, two players, / and B, choose integers n
1
, n
2
, n
3
,
. . . alternately according to the following rules:
Knowing n
2k
, / chooses any integer n
2k+1
such that
n
2k
n
2k+1
n
2
2k
.
Knowing n
2k+1
, B chooses any integer n
2k+2
such that
n
2k+1
n
2k+2
1.32. 32
ND
IMO, SIGTUNA, SWEDEN, 1991 37
is a prime raised to a positive integer power.
Player / wins the game by choosing the number 1990; player B wins by choosing the
number 1. For which n
0
does:
(a) / have a winning strategy?
(b) B have a winning strategy?
(c) Neither player have a winning strategy?
6. Prove that there exists a convex 1990-gon with the following two properties:
(a) All angles are equal.
(b) The lengths of the 1990 sides are the numbers 1
2
, 2
2
, 3
2
, . . . , 1990
2
in some
order.
1.32 32
nd
IMO, Sigtuna, Sweden, 1991
1. Given a triangle ABC, let I be the center of its inscribed circle. The internal
bisectors of the angles A, B, C meet the opposite sides in A

, B

, C

respectively.
Prove that
1
4
<
AI BI CI
AA

BB

CC


8
27
2. Let n > 6 be an integer and a
1
, a
2
, . . . , a
k
be all the natural numbers less than n
and relatively prime to n. If
a
2
a
1
= a
3
a
2
= = a
k
a
k1
> 0
prove that n must be either a prime number or a power of 2.
3. Let S = 1, 2, 3, . . . , 280. Find the smallest integer n such that each n-element
subset of S contains ve numbers which are pairwise relatively prime.
4. Suppose G is a connected graph with k edges. Prove that it is possible to label
the edges 1, 2, . . . , k in such a way that at each vertex which belongs to two or more
edges, the greatest common divisor of the integers labeling those edges is equal to 1.
[A graph consists of a set of points, called vertices, together with a set of edges
joining certain pairs of distinct vertices. Each pair of vertices u, v belongs to at
most one edge. The graph G is connected if for each pair of distinct vertices x, y
there is some sequence of vertices x = v
0
, v
1
, v
2
, . . . , v
m
= y such that each pair
v
i
, v
i+1
(0 i < m) is joined by an edge of G.]
38 CHAPTER 1. INTERNATIONAL MATHEMATICS OLYMPIAD
5. Let ABC be a triangle and P an interior point of ABC. Show that at least one
of the angles PAB, PBC, PCA is less than or equal to 30

.
6. An innite sequence x
0
, x
1
, x
2
, . . . of real numbers is said to be bounded if there is a
constant C such that [x
i
[ C for every i 0.
Given any real number a > 1, construct a bounded innite sequence x
0
, x
1
, x
2
, . . .
such that
[x
i
x
j
[[i j[
a
1
for every pair of distinct nonnegative integers i, j.
1.33 33
rd
IMO, Moscow, Russia, 1992
1. Find all integers a, b, c with 1 < a < b < c such that
(a 1)(b 1)(c 1) is a divisor of abc 1
2. Let R denote the set of all real numbers. Find all functions f : R R such that
f
_
x
2
+f(y)
_
= y + (f(x))
2
forall x, y R
3. Consider nine points in space, no four of which are coplanar. Each pair of points is
joined by an edge (that is, a line segment) and each edge is either colored blue or red
or left uncolored. Find the smallest value of n such that whenever exactly n edges
are colored, the set of colored edges necessarily contains a triangle all of whose edges
have the same color.
4. In the plane let C be a circle, L a line tangent to the circle C, and M a point
on L. Find the locus of all points P with the following property: there exists two
points Q, R on L such that M is the midpoint of QR and C is the inscribed circle
of triangle PQR.
5. Let S be a nite set of points in three-dimensional space. Let S
x
, S
y
, S
z
be the
sets consisting of the orthogonal projections of the points of S onto the yz-plane,
zx-plane, xy -plane, respectively. Prove that
[S[
2
[S
x
[ [S
y
[ [S
z
[
where [A[ denotes the number of elements in the nite set [A[. (Note: The orthogonal
projection of a point onto a plane is the foot of the perpendicular from that point to
the plane.)
1.34. 34
TH
IMO, ISTAMBUL, TURKEY, 1993 39
6. For each positive integer n, S(n) is dened to be the greatest integer such that, for
every positive integer k S(n), n
2
can be written as the sum of k positive squares.
(a) Prove that S(n) n
2
14 for each n 4.
(b) Find an integer n such that S(n) = n
2
14.
(c) Prove that there are inntely many integers n such that S(n) = n
2
14.
1.34 34
th
IMO, Istambul, Turkey, 1993
1. Let f(x) = x
n
+ 5x
n1
+ 3, where n > 1 is an integer. Prove that f(x) cannot be
expressed as the product of two nonconstant polynomials with integer coecients.
2. Let D be a point inside acute triangle ABC such that ADB = ACB + /2 and
AC BD = AD BC.
(a) Calculate the ratio (AB CD)/(AC BD).
(b) Prove that the tangents at C to the circumcircles of ACD and BCD are
perpendicular.
3. On an innite chessboard, a game is played as follows. At the start, n
2
pieces are
arranged on the chessboard in an n by n block of adjoining squares, one piece in each
square. A move in the game is a jump in a horizontal or vertical direction over an
adjacent occupied square to an unoccupied square immediately beyond. The piece
which has been jumped over is removed.
Find those values of n for which the game can end with only one piece remaining on
the board.
4. For three points P, Q, R in the plane, we dene m(PQR) as the minimum length of
the three altitudes of PQR. (If the points are collinear, we set m(PQR) = 0.)
Prove that for points A, B, C, X in the plane,
m(ABC) m(ABX) +m(AXC) +m(XBC)
5. Does there exist a function f : N N such that f(1) = 2, f(f(n)) = f(n) + n for
all n N, and f(n) < f(n + 1) for all n N?
40 CHAPTER 1. INTERNATIONAL MATHEMATICS OLYMPIAD
6. There are n lamps L
0
, . . . , L
n1
in a circle (n > 1), where we denote L
n+k
= L
k
. (A
lamp at all times is either on or o.) Perform steps s
0
, s
1
, . . . as follows: at step s
i
, if
L
i1
is lit, switch L
i
from on to o or vice versa, otherwise do nothing. Initially all
lamps are on. Show that:
(a) There is a positive integer M(n) such that after M(n) steps all the lamps are
on again;
(b) If n = 2
k
, we can take M(n) = n
2
1;
(c) If n = 2
k
+ 1, we can take M(n) = n
2
n + 1.
1.35 35
th
IMO, Hong Kong, 1994
1. Let mand n be positive integers. Let a
1
, a
2
, . . . , a
m
be distinct elements of 1, 2, . . . , n
such that whenever a
i
+a
j
n for some i, j, 1 i j m, there exists k, 1 k m,
with a
i
+a
j
= a
k
. Prove that
a
1
+a
2
+ +a
m
m

n + 1
2
2. ABC is an isosceles triangle with AB = AC. Suppose that
(a) M is the midpoint of BC and O is the point on the line AM such that OB is
perpendicular to AB;
(b) Q is an arbitrary point on the segment BC dierent from B and C;
(c) E lies on the line AB and F lies on the line AC such that E , Q, F are distinct
and collinear.
Prove that OQ is perpendicular to EF if and only if QE = QF.
3. For any positive integer k, let f(k) be the number of elements in the set k + 1, k +
2, . . . , 2k whose base 2 representation has precisely three 1s.
(a) Prove that, for each positive integer m, there exists at least onepositive integer
k such that f(k) = m.
(b) Determine all positive integers m for which there exists exactly one k with
f(k) = m.
1.36. 36
TH
IMO, TORONTO, CANADA, 1995 41
4. Determine all ordered pairs (m, n) of positive integers such that
n
3
+ 1
mn 1
is an integer.
5. Let S be the set of real numbers strictly greater than 1. Find all functions f : S S
satisfying the two conditions:
(a) f(x +f(y) +xf(y)) = y +f(x) +yf(x) for all x and y in S;
(b)
f(x)
x
is strictly increasing on each of the intervals 1 < x < 0 and 0 < x.
6. Show that there exists a set A of positive integers with the following property: For
any innite set S of primes there exist two positive integers m A and n / A each
of which is a product of k distinct elements of S for some k 2.
1.36 36
th
IMO, Toronto, Canada, 1995
1. Let A, B, C, D be four distinct points on a line, in that order. The circles with
diameters AC and BD intersect at X and Y . The line XY meets BC at Z. Let
P be a point on the line XY other than Z. The line CP intersects the circle with
diameter AC at C and M, and the line BP intersects the circle with diameter BD
at B and N. Prove that the lines AM, DN, XY are concurrent.
2. Let a, b, c be positive real numbers such that abc = 1. Prove that
1
a
3
(b +c)
+
1
b
3
(c +a)
+
1
c
3
(a +b)

3
2
3. Determine all integers n > 3 for which there exist n points A
1
, . . . , A
n
in the plane,
no three collinear, and real numbers r
1
, . . . , r
n
such that for 1 i < j < k n, the
area of A
i
A
j
A
k
is r
i
+r
j
+r
k
.
4. Find the maximum value of x
0
for which there exists a sequence x
0
, x
1
. . . , x
1995
of
positive reals with x
0
= x
1995
, such that for i = 1, . . . , 1995,
x
i1
+
2
x
i1
= 2x
i
+
1
x
i
42 CHAPTER 1. INTERNATIONAL MATHEMATICS OLYMPIAD
5. Let ABCDEF be a convex hexagon with AB = BC = CD and DE = EF = FA,
such that BCD = EFA = /3. Suppose G and H are points in theinterior of the
hexagon such that AGB = DHE = 2/3. Provethat AG + GB + GH + DH +
HE CF.
6. Let p be an odd prime number. How many p-element subsets A of 1, 2, . . . 2p are
there, the sum of whose elements is divisible by p?
1.37 37
th
IMO, Mumbai, India, 1996
1. We are given a positive integer r and a rectangular board ABCD with dimensions
[AB[ = 20, [BC[ = 12. The rectangle is divided into a grid of 20 12 unit squares.
The following moves are permitted on the board: one can move from one square to
another only if the distance between the centers of the two squares is

r. The task is
to nd a sequence of moves leading from the square with A as a vertex to the square
with B as a vertex.
(a) Show that the task cannot be done if r is divisible by 2 or 3.
(b) Prove that the task is possible when r = 73.
(c) Can the task be done when r = 97?
2. Let P be a point inside triangle ABC such that
APB ACB = APC ABC
Let D, E be the incenters of triangles APB, APC, respectively. Show that AP, BD, CE
meet at a point.
3. Let S denote the set of nonnegative integers. Find all functions f from S to itself
such that
f(m+f(n)) = f(f(m)) +f(n) m, n S
4. The positive integers a and b are such that the numbers 15a +16b and 16a 15b are
both squares of positive integers. What is the least possible value that can be taken
on by the smaller of these two squares?
5. Let ABCDEF be a convex hexagon such that AB is parallel to DE, BC is parallel
to EF, and CD is parallel to FA. Let R
A
, R
C
, R
E
denote the circumradii of trian-
gles FAB, BCD, DEF, respectively, and let P denote the perimeter of the hexagon.
1.38. 38
TH
IMO, MAR DEL PLATA, ARGENTINA, 1997 43
Prove that
R
A
+R
C
+R
E

P
2
6. Let p, q, n be three positive integers with p+q < n. Let (x
0
, x
1
, . . . , x
n
) be an (n+1)-
tuple of integers satisfying the following conditions:
(a) x
0
= x
n
= 0.
(b) For each i with 1 i n, either x
i
x
i1
= p or x
i
x
i1
= q.
Show that there exist indices i < j with (i, j) ,= (0, n), such that x
i
= x
j
.
1.38 38
th
IMO, Mar del Plata, Argentina, 1997
1. In the plane the points with integer coordinates are the vertices of unit squares. The
squares are colored alternately black and white (as on a chessboard). For any pair of
positive integers m and n, consider a right-angled triangle whose vertices have integer
coordinates and whose legs, of lengths m and n,ie along edges of the squares.
Let S
1
be the total area of the black part of the triangle and S
2
be the total area of
thehite part. Let
f(m, n) = [S
1
S
2
[
(a) Calculate f(m, n) for all positive integers m and n which are eitheroth even or
both odd.
(b) Prove that f(m, n)
1
2
maxm, n for all m and n.
(c) Show that there is no constant C such that f(m, n) < C for all m and n.
endenumerate
2. The angle at A is the smallest angle of triangle ABC. The points B and C divide
the circumcircle of the triangle into two arcs. Let U be an interior point of the arc
between B and C which does not contain A. The perpendicular bisectors of AB and
AC meet the line AU at V and W, respectively. The lines BV and CW meet at T.
Show that
AU = TB +TC
3. Let x
1
, x
2
, . . . , x
n
be real numbers satisfying the conditions
[x
1
+x
2
+ +x
n
[ = 1
44 CHAPTER 1. INTERNATIONAL MATHEMATICS OLYMPIAD
and
[x
i
[
n + 1
2
i = 1, 2, . . . , n
Show that there exists a permutation y
1
, y
2
, . . . , y
n
of x
1
, x
2
, . . . , x
n
such that
[y
1
+ 2y
2
+ +ny
n
[
n + 1
2
4. An n n matrix whose entries come from the set S = 1, 2, . . . , 2n 1 is called
a silver matrix if, for each i = 1, 2, . . . , n, the ith row and the ith column together
contain all elements of S. Show that
(a) there is no silver matrix for n = 1997;
(b) silver matrices exist for innitely many values of n.
5. Find all pairs (a, b) of integers a, b 1 that satisfy the equation
a
b
2
= b
a
6. For each positive integer n , let f(n) denote the number of ways of representing n as
a sum of powers of 2 with nonnegative integer exponents.epresentations which dier
only in the ordering of their summands are considered to be the same. For instance,
f(4) = 4, because the number 4 can be represented in the following four ways:
4; 2 + 2; 2 + 1 + 1; 1 + 1 + 1 + 1.
Prove that, for any integer n 3,
2
n
2
/4
< f(2
n
) < 2
n
2
/2
.
1.39 39
th
IMO, Taipei, Taiwan, 1998
1. In the convex quadrilateral ABCD, the diagonals AC and BD are perpendicular and
the opposite sides AB and DC are not parallel. Suppose that the point P, where the
perpendicular bisectors of AB and DC meet, is inside ABCD. Prove that ABCD is
a cyclic quadrilateral if and only if the triangles ABP and CDP have equal areas.
2. In a competition, there are a contestants and b judges, where b 3 is an odd integer.
Each judge rates each contestant as either pass or fail. Suppose k is a number such
that, for any two judges, their ratings coincide for at most k contestants. Prove that
k/a (b 1)/(2b).
1.40. 40
TH
IMO, BUCHAREST, ROMANIA, 1999 45
3. For any positive integer n, let d(n) denote the number of positive divisors of n (in-
cluding 1 and n itself). Determine all positive integers k such that d(n
2
)/d(n) = k
for some n.
4. Determine all pairs (a, b) of positive integers such that ab
2
+b +7 divides a
2
b +a +b.
5. Let I be the incenter of triangle ABC. Let the incircle of ABC touch the sides BC,
CA, and AB at K, L, and M, respectively. The line through B parallel to MK
meets the lines LM and LK at R and S, respectively. Prove that angle RIS is acute.
6. Consider all functions f from the set N of all positive integers into itself satisfying
f(t
2
f(s)) = s(f(t))
2
for all s and t in N. Determine the least possible value of
f(1998).
1.40 40
th
IMO, Bucharest, Romania, 1999
1. Determine all nite sets S of at least three points in the plane which satisfy the
following condition:
for any two distinct points A and B in S, the perpendicular bisector of the
line segment AB is an axis of symmetry for S.
2. Let n be a xed integer, with n 2.
(a) Determine the least constant C such that the inequality

1i<jn
x
i
x
j
(x
2
i
+x
2
j
) C
_
_

1in
x
i
_
_
4
holds for all real numbers x
1
, , x
n
0.
(b) For this constant C, determine when equality holds.
3. Consider an n n square board, where n is a xed even positive integer. The board
is divided into n
2
unit squares. We say that two dierent squares on the board are
adjacent if they have a common side.
N unit squares on the board are marked in such a way that every square (marked or
unmarked) on the board is adjacent to at least one marked square.
Determine the smallest possible value of N.
46 CHAPTER 1. INTERNATIONAL MATHEMATICS OLYMPIAD
4. Determine all pairs (n, p) of positive integers such that
p is a prime,
n not exceeded 2p, and
(p 1)
n
+ 1 is divisible by n
p1
.
5. Two circles G
1
and G
2
are contained inside the circle G, and are tangent to G at the
distinct points M and N, respectively. G
1
passes through the center of G
2
. The line
passing through the two points of intersection of G
1
and G
2
meets G at A and B.
The lines MA and MB meet G
1
at C and D, respectively.
Prove that CD is tangent to G
2
.
6. Determine all functions f : R R such that
f(x f(y)) = f(f(y)) +xf(y) +f(x) 1
for all real numbers x, y.
1.41 41
st
IMO, Taejon, South Korea, 2000
1. Two circles
1
and
2
intersect at M and N. Line is tangent to the circles at A
and B, respectively, so that M lies closer to than N. Line CD, with C on
1
and
D on
2
, is parallel to and passes through M. Let lines AC and BD meet at E;
let lines AN and CD meet at P; and let lines BN and CD meet at Q. Prove that
EP = EQ.
2. Let a, b, c be positive real numbers such that abc = 1. Prove that:
_
a 1 +
1
b
__
b 1 +
1
c
__
c 1 +
1
a
_
1
3. Let n 2 be a positive integer. Initially, there are n eas on a horizontal line, not
all at the same point.
For a positive real number , dene a move as follows:
choose any two eas at points A and B, with A to the left of B; let the ea
at A jump to the point C on the line to the right of B with
BC
AB
= .
Determine all the values of such that for any point M on the line and any initial
position of the n eas, there is a sequence of moves that will take all the eas to the
position to the right of M.
1.42. 42
ND
IMO, WASHINGTON DC, USA, 2001 47
4. A magician has one hundred cards numbered 1 to 100. He puts them into three boxes,
a red one, a white one and a blue one, so that each box contain at least one card.
A member of the audience selects two of the three boxes, choose one card from each
and announces the sum of the numbers on the chosen cards. Given this sum, the
magician identies the box from which no card has been choosen.
How many ways are there to put all the cards into the boxes so that this trick always
works? (Two ways are considered diferent if at least one of card is put nto a dierent
box)
5. Determine whether or not there exists a positive integer n such that:
n is divisible by exactely 2000 dierent prime numbers, and
2
n
+ 1 is divisible by n.
6. Let AH
1
, BH
2
, CH
3
be the altitudes of an acute-angled triangle ABC. The incircle
of the triangle ABC touches the sides BC, CA, AB at T
1
, T
2
, T
3
, respectively. Let
the lines
1
,
2
,
3
be the reections of the lines H
2
H
3
, H
3
H
1
H
1
H
2
in the lines T
2
T
3
,
T
3
T
1
, T
1
T
2
, respectively.
Prove that
1
,
2
,
3
determine a triangle whose vertices lie on the incircle of the
triangle ABC.
1.42 42
nd
IMO, Washington DC, USA, 2001
1. Let ABC be an acute-angled triangle with circumcentre O. Let P on BC be the
foot of the altitude from A. Suppose that BCA ABC + 30

. Prove that
CAB +COP < 90

2. Prove that
a

a
2
+ 8bc
+
b

b
2
+ 8ca
+
c

c
2
+ 8ab
1
for all positive real numbers a, b and c.
3. Twenty-one girls and twenty-one boys took part in a mathematical contest.
Each contestant solved at most six problems.
For each girl and each boy, at least one problem was solved by both of them
Prove that there was a problem that was solved by at least three girls and at least
three boys.
48 CHAPTER 1. INTERNATIONAL MATHEMATICS OLYMPIAD
4. Let n be an odd integer greater than 1, and let k
1
, k
2
, . . . , k
n
be given integers. For
each of the n! permutations a = (a
1
, a
2
, . . . , a
n
) of 1, 2, . . . , n, let
S (a) =
n

i=1
k
i
a
i
Prove that there are two permutations b and c, such that n! is a divisor of S (b)S (c).
5. In a triangle ABC, let AP bisect BAC, with P on BC, and let BQ bisect ABC,
with Q on CA. It is known that BAC = 60

and that AB + BP = AQ + QB.


What are the possible angles of triangle ABC?
6. Let a, b, c, d be integers with a > b > c > d > 0. Suppose that
ac +bd = (b +d +a c) (b +d a +c)
Prove ab +cd is not a prime.
1.43 43
rd
IMO, Glascow, United Kingdom, 2002
1. S is the set of all (h, k) with h, k non-negative integers such that h + k < n. Each
element of S is colored red or blue, so that if (h, k) is red and h

h, k

k, then
(h

, k

) is also red. A type 1 subset of S has n blue elements with dierent rst
member and a type 2 subset of S has n blue elements with dierent second member.
Show that there are the same number of type 1 and type 2 subsets.
2. BC is a diameter of a circle center O. A is any point on the circle with AOC >
60
o
. EF is the chord which is the perpendicular bisector of AO. D is the midpoint of
the minor arc AB. The line through O parallel to AD meets AC at J. Show that J
is the incenter of triangle CEF.
3. Find all pairs of integers m > 2, n > 2 such that there are innitely many positive
integers k for which k
n
+k
2
1 divides k
m
+k 1.
4. The positive divisors of the integer n > 1 are d
1
< d
2
< . . . < d
k
, so that d
1
= 1, d
k
=
n. Let d = d
1
d
2
+d
2
d
3
+ +d
k1
d
k
.
(a) Prove that D < n
2
.
(b) Determine all n for which D is a divisor of n
2
.
1.44. 44
TH
IMO, TOKYO, JAPAN, 2003 49
5. Find all functions f from the set R of real numbers to itself such that
(f(x) +f(z))(f(y) +f(t)) = f(xy zt) +f(xt +yz)
for all x, y, z, t in R.
6. n > 2 circles of radius 1 are drawn in the plane so that no line meets more than two
of the circles. Their centers are O
1
, O
2
, , O
n
. Show that

1i<jn
1
O
i
O
j

(n1)
4
1.44 44
th
IMO, Tokyo, Japan, 2003
1. Let A be a 101-element subset of the set S = 1, 2, . . . , 1000000. Prove that there
exist numbers t
1
, t
2
, . . . , t
100
in S such that the sets
A
j
= x +t
j
[ x A j = 1, 2, . . . , 100
are pairwise disjoint.
2. Find all pairs (m, n) of positive integers such that
m
2
2mn
2
n
3
+1
is a positive integer.
3. A convex hexagon is given in which any two opposite sides have the following property:
the distance between their midpoints is

3/2 times the sum of their lengths. Prove


that all the angles of the hexagon are equal.
(A convex ABCDEF has three pairs of opposite sides: AB and DE, BC and EF,
CD and FA.)
4. Let ABCD be a cyclic quadrilateral. Let P, Q and R be the feet of perpendiculars
from D to lines BC, CA and AB, respectively. Showhat PQ = QR if and only if the
bisectors of angles ABC and ADC meet on segment AC.
5. Let n be a positive integer and x
1
, x
2
, . . . , x
n
be real numbers with x
1
x
2
. . . x
n
.
(a) Prove that
_
_
n

i=1
n

j=1
[x
i
x
j
[
_
_
2

2(n
2
1)
3
n

i=1
n

j=1
(x
i
x
j
)
2
(b) Show that the equality holds if and only if x
1
, x
2
, . . . , x
n
form an arithmetic
sequence.
6. Show that for each prime p, there exists a prime q such that n
p
p is not divisible
by q for any positive integer n.
Chapter 2
William Lowell Putnam Competition
2.1 46
th
Anual William Lowell Putnam Competition,
1985
1. Determine, with proof, the number of ordered triples (A
1
, A
2
, A
3
) of sets which have
the property that
(i) A
1
A
2
A
3
= 1, 2, 3, 4, 5, 6, 7, 8, 9, 10, and
(ii) A
1
A
2
A
3
= .
Express your answer in the form 2
a
3
b
5
c
7
d
, where a, b, c, d are nonnegative integers.
2. Let T be an acute triangle. Inscribe a rectangle R in T with one side along a side of
T. Then inscribe a rectangle S in the triangle formed by the side of R opposite the
side on the boundary of T, and the other two sides of T, with one side along the side
of R. For any polygon X, let A(X) denote the area of X. Find the maximum value,
or show that no maximum exists, of
A(R)+A(S)
A(T)
, where T ranges over all triangles and
R, S over all rectangles as above.
3. Let d be a real number. For each integer m 0, dene a sequence a
m
(j), j =
0, 1, 2, . . . by the condition
a
m
(0) = d/2
m
,
a
m
(j + 1) = (a
m
(j))
2
+ 2a
m
(j), j 0.
Evaluate lim
n
a
n
(n).
50
2.1. 46
TH
ANUAL WILLIAM LOWELL PUTNAM COMPETITION, 1985 51
4. Dene a sequence a
i
by a
1
= 3 and a
i+1
= 3
a
i
for i 1. Which integers between
00 and 99 inclusive occur as the last two digits in the decimal expansion of innitely
many a
i
?
5. Let I
m
=
_
2
0
cos(x) cos(2x) cos(mx) dx. For which integers m, 1 m 10 is
I
m
,= 0?
6. If p(x) = a
0
+a
1
x + +a
m
x
m
is a polynomial with real coecients a
i
, then set
(p(x)) = a
2
0
+a
2
1
+ +a
2
m
.
Let F(x) = 3x
2
+ 7x + 2. Find, with proof, a polynomial g(x) with real coecients
such that
(i) g(0) = 1, and
(ii) (f(x)
n
) = (g(x)
n
)
for every integer n 1.
7. Let k be the smallest positive integer for which there exist distinct integers m
1
, m
2
, m
3
, m
4
, m
5
such that the polynomial
p(x) = (x m
1
)(x m
2
)(x m
3
)(x m
4
)(x m
5
)
has exactly k nonzero coecients. Find, with proof, a set of integers m
1
, m
2
, m
3
, m
4
, m
5
for which this minimum k is achieved.
8. Dene polynomials f
n
(x) for n 0 by f
0
(x) = 1, f
n
(0) = 0 for n 1, and
d
dx
f
n+1
(x) = (n + 1)f
n
(x + 1)
for n 0. Find, with proof, the explicit factorization of f
100
(1) into powers of distinct
primes.
9. Let
a
1,1
a
1,2
a
1,3
. . .
a
2,1
a
2,2
a
2,3
. . .
a
3,1
a
3,2
a
3,3
. . .
.
.
.
.
.
.
.
.
.
.
.
.
be a doubly innite array of positive integers, and suppose each positive integer
appears exactly eight times in the array. Prove that a
m,n
> mn for some pair of
positive integers (m, n).
52 CHAPTER 2. WILLIAM LOWELL PUTNAM COMPETITION
10. Let C be the unit circle x
2
+y
2
= 1. A point p is chosen randomly on the circumference
C and another point q is chosen randomly from the interior of C (these points are
chosen independently and uniformly over their domains). Let R be the rectangle with
sides parallel to the x and y-axes with diagonal pq. What is the probability that no
point of R lies outside of C?
11. Evaluate
_

0
t
1/2
e
1985(t+t
1
)
dt. You may assume that
_

e
x
2
dx =

.
12. Let G be a nite set of real n n matrices M
i
, 1 i r, which form a group
under matrix multiplication. Suppose that

r
i=1
tr(M
i
) = 0, where tr(A) denotes the
trace of the matrix A. Prove that

r
i=1
M
i
is the n n zero matrix.
2.2 47
th
Anual William Lowell Putnam Competition,
1986
1. Find, with explanation, the maximum value of f(x) = x
3
3x on the set of all real
numbers x satisfying x
4
+ 36 13x
2
.
2. What is the units (i.e., rightmost) digit of
_
10
20000
10
100
+ 3
_
?
3. Evaluate

n=0
Arccot(n
2
+n + 1), where Arccot t for t 0 denotes the number in
the interval 0 < /2 with cot = t.
4. A transversal of an nn matrix A consists of n entries of A, no two in the same row
or column. Let f(n) be the number of n n matrices A satisfying the following two
conditions:
(a) Each entry
i,j
of A is in the set 1, 0, 1.
(b) The sum of the n entries of a transversal is the same for all transversals of A.
An example of such a matrix A is
A =
_
_
_
1 0 1
0 1 0
0 1 0
_
_
_.
2.2. 47
TH
ANUAL WILLIAM LOWELL PUTNAM COMPETITION, 1986 53
Determine with proof a formula for f(n) of the form
f(n) = a
1
b
n
1
+a
2
b
n
2
+a
3
b
n
3
+a
4
,
where the a
i
s and b
i
s are rational numbers.
5. Suppose f
1
(x), f
2
(x), . . . , f
n
(x) are functions of n real variables x = (x
1
, . . . , x
n
) with
continuous second-order partial derivatives everywhere on R
n
. Suppose further that
there are constants c
ij
such that
f
i
x
j

f
j
x
i
= c
ij
for all i and j, 1 i n, 1 j n. Prove that there is a function g(x) on R
n
such
that f
i
+g/x
i
is linear for all i, 1 i n. (A linear function is one of the form
a
0
+a
1
x
1
+a
2
x
2
+ +a
n
x
n
.)
6. Let a
1
, a
2
, . . . , a
n
be real numbers, and let b
1
, b
2
, . . . , b
n
be distinct positive integers.
Suppose that there is a polynomial f(x) satisfying the identity
(1 x)
n
f(x) = 1 +
n

i=1
a
i
x
b
i
.
Find a simple expression (not involving any sums) for f(1) in terms of b
1
, b
2
, . . . , b
n
and n (but independent of a
1
, a
2
, . . . , a
n
).
7. Inscribe a rectangle of base b and height h in a circle of radius one, and inscribe
an isosceles triangle in the region of the circle cut o by one base of the rectangle
(with that side as the base of the triangle). For what value of h do the rectangle and
triangle have the same area?
8. Prove that there are only a nite number of possibilities for the ordered triple T =
(x y, y z, z x), where x, y, z are complex numbers satisfying the simultaneous
equations
x(x 1) + 2yz = y(y 1) + 2zx +z(z 1) + 2xy,
and list all such triples T.
9. Let consist of all polynomials in x with integer coecienst. For f and g in and
m a positive integer, let f g (mod m) mean that every coecient of f g is an
integral multiple of m. Let n and p be positive integers with p prime. Given that
f, g, h, r and s are in with rf + sg 1 (mod p) and fg h (mod p), prove
that there exist F and G in with F f (mod p), G g (mod p), and FG h
(mod p
n
).
54 CHAPTER 2. WILLIAM LOWELL PUTNAM COMPETITION
10. For a positive real number r, let G(r) be the minimum value of [r

m
2
+ 2n
2
[ for
all integers m and n. Prove or disprove the assertion that lim
r
G(r) exists and
equals 0.
11. Let f(x, y, z) = x
2
+y
2
+z
2
+xyz. Let p(x, y, z), q(x, y, z), r(x, y, z) be polynomials
with real coecients satisfying
f(p(x, y, z), q(x, y, z), r(x, y, z)) = f(x, y, z).
Prove or disprove the assertion that the sequence p, q, r consists of some permutation
of x, y, z, where the number of minus signs is 0 or 2.
12. Suppose A, B, C, D are n n matrices with entries in a eld F, satisfying the condi-
tions that AB
T
andCD
T
are symmetric and AD
T
BC
T
= I. Here I is the n n
identity matrix, and if M is an n n matrix, M
T
is its transpose. Prove that
A
T
D +C
T
B = I.
2.3 48
th
Anual William Lowell Putnam Competition,
1987
1. Curves A, B, C and D are dened in the plane as follows:
A =
_
(x, y) : x
2
y
2
=
x
x
2
+y
2
_
,
B =
_
(x, y) : 2xy +
y
x
2
+y
2
= 3
_
,
C =
_
(x, y) : x
3
3xy
2
+ 3y = 1
_
,
D =
_
(x, y) : 3x
2
y 3x y
3
= 0
_
.
Prove that A B = C D.
2. The sequence of digits
123456789101112131415161718192021 . . .
is obtained by writing the positive integers in order. If the 10
n
-th digit in this sequence
occurs in the part of the sequence in which the m-digit numbers are placed, dene
f(n) to be m. For example, f(2) = 2 because the 100th digit enters the sequence in
the placement of the two-digit integer 55. Find, with proof, f(1987).
2.3. 48
TH
ANUAL WILLIAM LOWELL PUTNAM COMPETITION, 1987 55
3. For all real x, the real-valued function y = f(x) satises
y

2y

+y = 2e
x
.
(a) If f(x) > 0 for all real x, must f

(x) > 0 for all real x? Explain.


(b) If f

(x) > 0 for all real x, must f(x) > 0 for all real x? Explain.
4. Let P be a polynomial, with real coecients, in three variables and F be a function
of two variables such that
P(ux, uy, uz) = u
2
F(y x, z x) for all real x, y, z, u,
and such that P(1, 0, 0) = 4, P(0, 1, 0) = 5, and P(0, 0, 1) = 6. Also let A, B, C be
complex numbers with P(A, B, C) = 0 and [B A[ = 10. Find [C A[.
5. Let

G(x, y) =
_
y
x
2
+ 4y
2
,
x
x
2
+ 4y
2
, 0
_
.
Prove or disprove that there is a vector-valued function

F(x, y, z) = (M(x, y, z), N(x, y, z), P(x, y, z))


with the following properties:
(i) M, N, P have continuous partial derivatives for all (x, y, z) ,= (0, 0, 0);
(ii) Curl

F =

0 for all (x, y, z) ,= (0, 0, 0);


(iii)

F(x, y, 0) =

G(x, y).
6. For each positive integer n, let a(n) be the number of zeroes in the base 3 represen-
tation of n. For which positive real numbers x does the series

n=1
x
a(n)
n
3
converge?
7. Evaluate
_
4
2
_
ln(9 x) dx
_
ln(9 x) +
_
ln(x + 3)
.
56 CHAPTER 2. WILLIAM LOWELL PUTNAM COMPETITION
8. Let r, s and t be integers with 0 r, 0 s and r +s t. Prove that
_
s
0
_
_
t
r
_
+
_
s
1
_
_
t
r+1
_
+ +
_
s
s
_
_
t
r+s
_
=
t + 1
(t + 1 s)
_
ts
r
_
9. Let F be a eld in which 1 + 1 ,= 0. Show that the set of solutions to the equation
x
2
+y
2
= 1 with x and y in F is given by (x, y) = (1, 0) and
(x, y) =
_
r
2
1
r
2
+ 1
,
2r
r
2
+ 1
_
where r runs through the elements of F such that r
2
,= 1.
10. Let (x
1
, y
1
) = (0.8, 0.6) and let x
n+1
= x
n
cos y
n
y
n
sin y
n
and y
n+1
= x
n
sin y
n
+
y
n
cos y
n
for n = 1, 2, 3, . . .. For each of lim
n
x
n
and lim
n
y
n
, prove that the
limit exists and nd it or prove that the limit does not exist.
11. Let O
n
be the n-dimensional vector (0, 0, , 0). Let M be a 2n n matrix of
complex numbers such that whenever (z
1
, z
2
, . . . , z
2n
)M = O
n
, with complex z
i
, not
all zero, then at least one of the z
i
is not real. Prove that for arbitrary real numbers
r
1
, r
2
, . . . , r
2n
, there are complex numbers w
1
, w
2
, . . . , w
n
such that
re
_

_
M
_
_
_
_
w
1
.
.
.
w
n
_
_
_
_
_

_
=
_
_
_
_
r
1
.
.
.
r
n
_
_
_
_
.
(Note: if C is a matrix of complex numbers, re(C) is the matrix whose entries are
the real parts of the entries of C.)
12. Let F be the eld of p
2
elements, where p is an odd prime. Suppose S is a set of
(p
2
1)/2 distinct nonzero elements of F with the property that for each a ,= 0 in F,
exactly one of a and a is in S. Let N be the number of elements in the intersection
S 2a : a S. Prove that N is even.
2.4 49
th
Anual William Lowell Putnam Competition,
1988
1. Let R be the region consisting of the points (x, y) of the cartesian plane satisfying
both [x[ [y[ 1 and [y[ 1. Sketch the region R and nd its area.
2.4. 49
TH
ANUAL WILLIAM LOWELL PUTNAM COMPETITION, 1988 57
2. A not uncommon calculus mistake is to believe that the product rule for derivatives
says that (fg)

= f

. If f(x) = e
x
2
, determine, with proof, whether there exists an
open interval (a, b) and a nonzero function g dened on (a, b) such that this wrong
product rule is true for x in (a, b).
3. Determine, with proof, the set of real numbers x for which

n=1
_
1
n
csc
1
n
1
_
x
converges.
4. (a) If every point of the plane is painted one of three colors, do there necessarily
exist two points of the same color exactly one inch apart?
(b) What if three is replaced by nine?
5. Prove that there exists a unique function f from the set R
+
of positive real numbers
to R
+
such that
f(f(x)) = 6x f(x)
and
f(x) > 0
for all x > 0.
6. If a linear transformation A on an n-dimensional vector space has n +1 eigenvectors
such that any n of them are linearly independent, does it follow that A is a scalar
multiple of the identity? Prove your answer.
7. A composite (positive integer) is a product ab with a and b not necessarily distinct
integers in 2, 3, 4, . . .. Show that every composite is expressible as xy +xz +yz +1,
with x, y, z positive integers.
8. Prove or disprove: If x and y are real numbers with y 0 and y(y + 1) (x + 1)
2
,
then y(y 1) x
2
.
9. For every n in the set N = 1, 2, . . . of positive integers, let r
n
be the minimum value
of [c d

3[ for all nonnegative integers c and d with c + d = n. Find, with proof,


the smallest positive real number g with r
n
g for all n N.
10. Prove that if

n=1
a
n
is a convergent series of positive real numbers, then so is

n=1
(a
n
)
n/(n+1)
.
58 CHAPTER 2. WILLIAM LOWELL PUTNAM COMPETITION
11. For positive integers n, let M
n
be the 2n + 1 by 2n + 1 skew-symmetric matrix for
which each entry in the rst n subdiagonals below the main diagonal is 1 and each
of the remaining entries below the main diagonal is -1. Find, with proof, the rank
of M
n
. (According to one denition, the rank of a matrix is the largest k such that
there is a k k submatrix with nonzero determinant.)
One may note that
M
1
=
_
_
_
0 1 1
1 0 1
1 1 0
_
_
_
M
2
=
_
_
_
_
_
_
_
_
0 1 1 1 1
1 0 1 1 1
1 1 0 1 1
1 1 1 0 1
1 1 1 1 0
_
_
_
_
_
_
_
_
12. Prove that there exist an innite number of ordered pairs (a, b) of integers such that
for every positive integer t, the number at +b is a triangular number if and only if t
is a triangular number. (The triangular numbers are the t
n
= n(n + 1)/2 with n in
0, 1, 2, . . .)
2.5 50
th
Anual William Lowell Putnam Competition,
1989
1. How many primes among the positive integers, written as usual in base 10, are alter-
nating 1s and 0s, beginning and ending with 1?
2. Evaluate
_
a
0
_
b
0
e
max{b
2
x
2
,a
2
y
2
}
dy dx where a and b are positive.
3. Prove that if
11z
10
+ 10iz
9
+ 10iz 11 = 0,
then [z[ = 1. (Here z is a complex number and i
2
= 1.)
4. If is an irrational number, 0 < < 1, is there a nite game with an honest coin
such that the probability of one player winning the game is ? (An honest coin is one
for which the probability of heads and the probability of tails are both
1
2
. A game is
nite if with probability 1 it must end in a nite number of moves.)
2.5. 50
TH
ANUAL WILLIAM LOWELL PUTNAM COMPETITION, 1989 59
5. Let m be a positive integer and let ( be a regular (2m+1)-gon inscribed in the unit
circle. Show that there is a positive constant A, independent of m, with the following
property. For any points p inside ( there are two distinct vertices v
1
and v
2
of ( such
that
[ [p v
1
[ [p v
2
[ [ <
1
m

A
m
3
.
Here [s t[ denotes the distance between the points s and t.
6. Let = 1 +a
1
x +a
2
x
2
+ be a formal power series with coecients in the eld of
two elements. Let
a
n
=
_

_
1 if every block of zeros in the binary expansion of n
has an even number of zeros in the block
0 otherwise.
(For example, a
36
= 1 because 36 = 100100
2
and a
20
= 0 because 20 = 10100
2
.)
Prove that
3
+x + 1 = 0.
7. A dart, thrown at random, hits a square target. Assuming that any two parts of the
target of equal area are equally likely to be hit, nd the probability that the point hit
is nearer to the center than to any edge. Express your answer in the form
a

b +c
d
,
where a, b, c, d are integers.
8. Let S be a non-empty set with an associative operation that is left and right can-
cellative (xy = xz implies y = z, and yx = zx implies y = z).ssume that for every a
in S the set a
n
: n = 1, 2, 3, . . . is inite. Must S be a group?
9. Let f be a function on [0, ), dierentiable and satisfying
f

(x) = 3f(x) + 6f(2x)


for x > 0. Assume that [f(x)[ e

x
for x 0 (so that f(x) tends rapidly to 0 as x
increases). For n a non-negative integer, dene

n
=
_

0
x
n
f(x) dx
(sometimes called the nth moment of f).
a) Express
n
in terms of
0
.
b) Prove that the sequence
n
3
n
n!
always converges, and that the limit is 0 only
if
0
= 0.
60 CHAPTER 2. WILLIAM LOWELL PUTNAM COMPETITION
10. Can a countably innite set have an uncountable collection of non-empty subsets such
that the intersection of any two of them is nite?
11. Label the vertices of a trapezoid T (quadrilateral with two parallel sides) inscribed
in the unit circle as A, B, C, D so that AB is parallel to CD and A, B, C, D are
in counterclockwise order. Let s
1
, s
2
, and d denote the lengths of the line segments
AB, CD, and OE, where E is the point of intersection of the diagonals of T, and O
is the center of the circle. Determine the least upper bound of
s
1
s
2
d
over all such T
for which d ,= 0, and describe allases, if any, in which it is attained.
12. Let (x
1
, x
2
, . . . x
n
) be a point chosen at random from the n-dimensional region dened
by 0 < x
1
< x
2
< < x
n
< 1. Let f be a continuous function on [0, 1] with f(1) = 0.
Set x
0
= 0 and x
n+1
= 1. Show that the expected value of the Riemann sum
n

i=0
(x
i+1
x
i
)f(x
i+1
)
is
_
1
0
f(t)P(t) dt, where P is a polynomial of degree n, independent of f, with 0
P(t) 1 for 0 t 1.
2.6 51
th
Anual William Lowell Putnam Competition,
1990
1. Let
T
0
= 2, T
1
= 3, T
2
= 6,
and for n 3,
T
n
= (n + 4)T
n1
4nT
n2
+ (4n 8)T
n3
.
The rst few terms are
2, 3, 6, 14, 40, 152, 784, 5168, 40576.
Find, with proof, a formula for T
n
of the form T
n
= A
n
+B
n
, where A
n
and B
n

are well-known sequences.


2. Is

2 the limit of a sequence of numbers of the form


3

n
3

m (n, m = 0, 1, 2, . . .)?
3. Prove that any convex pentagon whose vertices (no three of which are collinear) have
integer coordinates must have area greater than or equal to
5
2
.
2.6. 51
TH
ANUAL WILLIAM LOWELL PUTNAM COMPETITION, 1990 61
4. Consider a paper punch that can be centered at any point of the plane and that,
when operated, removes from the plane precisely those points whose distance from
the center is irrational. How many punches are needed to remove every point?
5. If A and B are square matrices of the same size such that ABAB = 0, does it follow
that BABA = 0?
6. If X is a nite set, let X denote the number of elements in X. Call an ordered pair
(S, T) of subsets of 1, 2, . . . , n admissible if s > [T[ for each s S, and t > [S[ for
each t T. How many admissible ordered pairs of subsets of 1, 2, . . . , 10 are there?
Prove your answer.
7. Find all real-valued continuously dierentiable functions f on the real line such that
for all x,
(f(x))
2
=
_
x
0
[(f(t))
2
+ (f

(t))
2
] dt + 1990.
8. Prove that for [x[ < 1, [z[ > 1,
1 +

j=1
(1 +x
j
)P
j
= 0,
where P
j
is
(1 z)(1 zx)(1 zx
2
) (1 zx
j1
)
(z x)(z x
2
)(z x
3
) (z x
j
)
.
9. Let S be a set of 22 integer matrices whose entries a
ij
(1) are all squares of integers
and, (2) satisfy a
ij
200. Show that if S has more than 50387 (= 15
4
15
2
15 +2)
elements, then it has two elements that commute.
10. Let G be a nite group of order n generated by a and b. Prove or disprove: there is
a sequence
g
1
, g
2
, g
3
, . . . , g
2n
such that
(1) every element of G occurs exactly twice, and
(2) g
i+1
equals g
i
a or g
i
b for i = 1, 2, . . . , 2n. (Interpret g
2n+1
as g
1
.)
11. Is there an innite sequence a
0
, a
1
, a
2
, . . . of nonzero real numbers such that for n =
1, 2, 3, . . . the polynomial
p
n
(x) = a
0
+a
1
x +a
2
x
2
+ +a
n
x
n
has exactly n distinct real roots?
62 CHAPTER 2. WILLIAM LOWELL PUTNAM COMPETITION
12. Let S be a nonempty closed bounded convex set in the plane. Let K be a line and t
a positive number. Let L
1
and L
2
be support lines for S parallel to K
1
, and let L be
the line parallel to K and midway between L
1
and L
2
. Let B
S
(K, t) be the band of
points whose distance from L is at most (t/2)w, where w is the distance between L
1
and L
2
. What is the smallest t such that
S

K
B
S
(K, t) ,=
for all S? (K runs over all lines in the plane.)
2.7 52
th
Anual William Lowell Putnam Competition,
1991
1. A 23 rectangle has vertices as (0, 0), (2, 0), (0, 3), and (2, 3). It rotates 90

clockwise
about the point (2, 0). It then otates 90

clockwise about the point (5, 0), then 90

clockwise about the point (7, 0), and nally, 90

clockwise about the point (10, 0).


(The side originally on the x-axis is now back on the x-axis.) Find the area of the
region above the x-axis and below the curve traced out by the point whose initial
position is (1,1).
2. Let A and B be dierent nn matrices with real entries. If A
3
= B
3
and A
2
B = B
2
A,
can A
2
+B
2
be nvertible?
3. Find all real polynomials p(x) of degree n 2 for which there exist real numbers
r
1
< r
2
< < r
n
such that
(a) p(r
i
) = 0, i = 1, 2, . . . , n, and
(b) p

_
r
i
+r
i+1
2
_
= 0 i = 1, 2, . . . , n 1,
where p

(x) denotes the derivative of p(x).


4. Does there exist an innite sequence of closed discs D
1
, D
2
, D
3
, . . . in the plane, with
centers c
1
, c
2
, c
3
, . . ., respectively, such that
(a) the c
i
have no limit point in the nite plane,
(b) the sum of the areas of the D
i
is nite, and
(c) every line in the plane intersects at least one of the D
i
?
2.7. 52
TH
ANUAL WILLIAM LOWELL PUTNAM COMPETITION, 1991 63
5. Find the maximum value of
_
y
0
_
x
4
+ (y y
2
)
2
dx
for 0 y 1.
6. Let A(n) denote the number of sums of positive integers
a
1
+a
2
+ +a
r
which add up to n with
a
1
> a
2
+a
3
, a
2
> a
3
+a
4
, . . .
a
r2
> a
r1
+a
r
, a
r1
> a
r
Let B(n) denote the number of b
1
+b
2
+ +b
s
which add up to n, with
(a) b
1
b
2
. . . b
s
,
(b) each b
i
is in the sequence 1, 2, 4, . . . , g
j
, . . . dened by g
1
= 1, g
2
= 2, and
g
j
= g
j1
+g
j2
+ 1, and
(c) if b
1
= g
k
then every element in 1, 2, 4, . . . , g
k
appears at least once as a b
i
.
Prove that A(n) = B(n) for each n 1.
(For example, A(7) = 5 because the relevant sums are 7, 6 +1, 5 +2, 4 +3, 4 +2 +1,
and B(7) = 5 because the relevant sums are 4 + 2 + 1, 2 + 2 + 2 + 1, 2 + 2 + 1 + 1 +
1, 2 + 1 + 1 + 1 + 1 + 1, 1 + 1 + 1 + 1 + 1 + 1 + 1.)
7. For each integer n 0, let S(n) = n m
2
, where m is the greatest integer with
m
2
n. Dene a sequence (a
k
)

k=0
by a
0
= A and a
k+1
= a
k
+S(a
k
) for k 0. For
what positive integers A is this sequence eventually constant?
8. Suppose f and g are non-constant, dierentiable, real-valued functions dened on
(, ). Furthermore, suppose that for each pair of real numbers x and y,
f(x + y) = f(x)f(y) g(x)g(y),
g(x + y) = f(x)g(y) +g(x)f(y).
If f

(0) = 0, prove that (f(x))


2
+ (g(x))
2
= 1 for all x.
9. Does there exist a real number L such that, if m and n are integers greater than L,
then an m n rectangle may be expressed as a nion of 4 6 and 5 7 rectangles,
any two of which intersect at most along their boundaries?
64 CHAPTER 2. WILLIAM LOWELL PUTNAM COMPETITION
10. Suppose p is an odd prime. Prove that
p

j=0
_
p
j
__
p +j
j
_
2
p
+ 1 (mod p
2
).
11. Let p be an odd prime and let Z
p
denote (the eld of) integers modulo p. How many
elements are in the set
x
2
: x Z
p
y
2
+ 1 : y Z
p
?
12. Let a and b be positive numbers. Find the largest number c, in terms of a and b, such
that
a
x
b
1x
a
sinh ux
sinh u
+b
sinh u(1 x)
sinh u
for all u with 0 < [u[ c and for all x, 0 < x < 1. (Note: sinh u = (e
u
e
u
)/2.)
2.8 53
th
Anual William Lowell Putnam Competition,
1992
1. Prove that f(n) = 1 n is the only integer-valued function dened on the integers
that satises the following conditions.
(i) f(f(n)) = n, for all integers n;
(ii) f(f(n + 2) + 2) = n for all integers n;
(iii) f(0) = 1.
2. Dene C() to be the coecient of x
1992
in the power series about x = 0 of (1 +x)

.
Evaluate
_
1
0
_
C(y 1)
1992

k=1
1
y +k
_
dy.
3. For a given positive integer m, nd all triples (n, x, y) of positive integers, with n
relatively prime to m, which satisfy
(x
2
+y
2
)
m
= (xy)
n
.
2.8. 53
TH
ANUAL WILLIAM LOWELL PUTNAM COMPETITION, 1992 65
4. Let f be an innitely dierentiable real-valued function dened on the real numbers.
If
f
_
1
n
_
=
n
2
n
2
+ 1
, n = 1, 2, 3, . . . ,
compute the values of the derivatives f
(k)
(0), k = 1, 2, 3, . . ..
5. For each positive integer n, let a
n
= 0 (or 1) if the number of 1s in the binary
representation of n is even (or odd), respectively. Show that there do not exist
positive integers k and m such that
a
k+j
= a
k+m+j
= a
k+m+2j
,
for 0 j m1.
6. Four points are chosen at random on the surface of a sphere. What is the probability
that the center of the sphere lies inside the tetrahedron whose vertices are at the
four points? (It is understood that each point is independently chosen relative to a
uniform distribution on the sphere.)
7. Let S be a set of n distinct real numbers. Let A
S
be the set of numbers that occur
as averages of two distinct elements of S. For a given n 2, what is the smallest
possible number of elements in A
S
?
8. For nonnegative integers n and k, dene Q(n, k) to be the coecient of x
k
in the
expansion of (1 +x +x
2
+ x
3
)
n
. Prove that
Q(n, k) =
k

j=0
_
n
j
__
n
k 2j
_
,
where
_
a
b
_
is the standard binomial coecient. (Reminder: For integers a and b with
a 0,
_
a
b
_
=
a!
b!(ab)!
for 0 b a, with
_
a
b
_
= 0 otherwise.)
9. For any pair (x, y) of real numbers, a sequence (a
n
(x, y))
n0
is dened as follows:
a
0
(x, y) = x,
a
n+1
(x, y) =
(a
n
(x, y))
2
+y
2
2
, for n 0.
Find the area of the region
(x, y)[(a
n
(x, y))
n0
converges.
66 CHAPTER 2. WILLIAM LOWELL PUTNAM COMPETITION
10. Let p(x) be a nonzero polynomial of degree less than 1992 having no nonconstant
factor in common with x
3
x. Let
d
1992
dx
1992
_
p(x)
x
3
x
_
=
f(x)
g(x)
for polynomials f(x) and g(x). Find the smallest possible degree of f(x).
11. Let D
n
denote the value of the (n 1) (n 1) determinant
_

_
3 1 1 1 1
1 4 1 1 1
1 1 5 1 1
1 1 1 6 1
.
.
.
.
.
.
.
.
.
.
.
.
.
.
.
.
.
.
1 1 1 1 n + 1
_

_
.
Is the set
_
Dn
n!
_
n2
bounded?
12. Let M be a set of real n n matrices such that
(i) I M, where I is the n n identity matrix;
(ii) if A M and B M, then either AB M or AB M, but not both;
(iii) if A M and B M, then either AB = BA or AB = BA;
(iv) if A M and A ,= I, there is at least one B M such that AB = BA.
Prove that M contains at most n
2
matrices.
2.9 54
th
Anual William Lowell Putnam Competition,
1993
1. The horizontal line y = c intersects the curve y = 2x 3x
3
in the rst quadrant as
in the gure. Find c so that the areas of the two shaded regions are equal. [Figure
not included. The rst region is bounded by the y-axis, the line y = c and the curve;
the other lies under the curve and above the line y = c between their two points of
intersection.]
2. Let (x
n
)
n0
be a sequence of nonzero real numbers such that x
2
n
x
n1
x
n+1
= 1 for
n = 1, 2, 3, . . .. Prove there exists a real number a such that x
n+1
= ax
n
x
n1
for
all n 1.
2.9. 54
TH
ANUAL WILLIAM LOWELL PUTNAM COMPETITION, 1993 67
3. Let T
n
be the set of subsets of 1, 2, . . . , n. Let c(n, m) be the number of functions
f : T
n
1, 2, . . . , m such that f(A B) = minf(A), f(B). Prove that
c(n, m) =
m

j=1
j
n
.
4. Let x
1
, x
2
, . . . , x
19
be positive integers each of which is less than or equal to 93. Let
y
1
, y
2
, . . . , y
93
be positive integers each of which is less than or equal to 19. Prove
that there exists a (nonempty) sum of some x
i
s equal to a sum of some y
j
s.
5. Show that
_
10
100
_
x
2
x
x
3
3x + 1
_
2
dx +
_ 1
11
1
101
_
x
2
x
x
3
3x + 1
_
2
dx +
_ 11
10
101
100
_
x
2
x
x
3
3x + 1
_
2
dx
is a rational number.
6. The innite sequence of 2s and 3s
2, 3, 3, 2, 3, 3, 3, 2, 3, 3, 3, 2, 3, 3, 2, 3, 3, 3, 2, 3, 3, 3, 2, 3, 3, 3, 2, 3, 3, 2, 3, 3, 3, 2, . . .
as the property that, if one forms a second sequence that records the number of 3s
between successive 2s, the result is identical to the iven sequence. Show that there
exists a real number r such that, for ny n, the nth term of the sequence is 2 if and
only if n = 1 + rm| for some nonnegative integer m. (Note: xrfloor denotes the
largest integer less than or equal to x.)
7. Find the smallest positive integer n such that for every integer m with 0 < m < 1993,
there exists an integer k for which
m
1993
<
k
n
<
m + 1
1994
.
8. Consider the following game played with a deck of 2n cards numbered from 1 to 2n.
The deck is randomly shued and n cards are dealt to each of two players. Beginning
with A, the players take turns discarding one of their remaining cards and announcing
its number. The game ends as soon as the sum of the numbers on the discarded cards
is divisible by 2n + 1. The last person to discard wins the game. Assuming optimal
strategy by both A and B, what is the probability that A wins?
9. Two real numbers x and y are chosen at random in the interval (0,1) with respect
to the uniform distribution. What is the probability that he closest integer to x/y is
even? Express the answer in the form r +s, where r and s are rational numbers.
68 CHAPTER 2. WILLIAM LOWELL PUTNAM COMPETITION
10. The function K(x, y) is positive and continuous for 0 x 1, 0 y 1, and the
functions f(x) and g(x) are positive and continuous for 0 x 1. Suppose that for
all x, 0 x 1,
_
1
0
f(y)K(x, y) dy = g(x)
and
_
1
0
g(y)K(x, y) dy = f(x).
Show that f(x) = g(x) for 9 x 1.
11. Show there do not exist four points in the Euclidean plane such that the pairwise
distances between the points are all odd integers.
12. Let S be a set of three, not necessarily distinct, positive integers. Show that one can
transform S into a set containing 0 by a nite number of applications of the following
rule: Select two of the three integers, say x and y, where x < y and replace them
with 2x and y x.
2.10 55
th
Anual William Lowell Putnam Competition,
1994
1. Let (a
n
) be a sequence of positive reals such that, for all n, a
n
a
2n
+a
2n+1
. Prove
that

n=1
a
n
diverges.
2. Find the positive value of m such that the area in the rst quadrant enclosed by the
ellipse
x
2
9
+ y
2
= 1, the x-axis, and the line y = 2x/3 is equal to the area in the rst
quadrant enclosed by the ellipse
x
2
9
+y
2
= 1, the y-axis, and the line y = mx.
3. Prove that the points of an isosceles triangle of side length 1 annot be colored in four
colors such that no two points at distance at least 2

2 from each other receive the


same color.
4. Let A and B be 2 2 matrices with integer entries such that each of A, A +B, A +
2B, A + 3B, A + 4B has an inverse with integer entries. Prove that the same must
be true of A+ 5B.
5. Let (r
n
) be a sequence of positive reals with limit 0. Let S be the set of all numbers
expressible in the form r
i
1
+ . . . + r
i
1994
for positive integers i
1
< i
2
< . . . < i
1994
.
Prove that every interval (a, b) contains a subinterval (c, d) whose intersection with
S is empty.
2.11. 56
TH
ANUAL WILLIAM LOWELL PUTNAM COMPETITION, 1995 69
6. Let f
1
, . . . , f
10
be bijections of the integers such that for every integer n, there exists
a sequence i
1
, . . . , i
k
for some k such that f
i
1
. . . f
i
k
(0) = n. Prove that if A is
any nonempty nite set, there exist at most 512 sequences (e
1
, . . . , e
10
) of zeroes and
ones such that f
e
1
1
. . . f
e
10
10
maps A to A. (Here f
1
= f and f
0
means the identity
function.)
7. Find all positive integers n such that [n m
2
[ 250 for exactly 15 nonnegative
integers m.
8. Find all c such that the graph of the function x
4
+9x
3
+cx
2
+ax+b meets some line
in four distinct points.
9. Let f(x) be a positive-valued function over the reals such that f

(x) > f(x) for all x.


For what k must there exist N such that f(x) > e
kx
for x > N?
10. Let A be the matrix
_
3 2
4 2
_
and for positive integers n, dene d
n
as the greatest
common divisor of the entries of A
n
I, where I = ((10)(01)). Prove that d
n

as n .
11. Fix n a positive integer. For real, dene f

(i) as the greatest integer less than or


equal to i, and write f
k
for the k-th iterate of f (i.e. f
1
= f and f
k+1
= f f
k
).
Prove there exists such that f

k (n
2
) = f
k

(n
2
) = n
2
k for k = 1, . . . , n.
12. Suppose a, b, c, d are integers with 0 a bleq99, 0 c d 99. For any integer i,
let n
i
= 101i + 1002
i
. Show that if n
a
+ n
b
is congruent to n
c
+n
d
mod 10100, then
a = c and b = d.
2.11 56
th
Anual William Lowell Putnam Competition,
1995
1. Let S be a set of real numbers which is closed under multiplication (that is, if a and
b are in S, then so is ab). Let T and U be disjoint subsets of S whose union is S.
Given that the product of any three (not necessarily distinct) elements of T is in T
and that the product of any three elements of U is in U, show that at least one of
the two subsets T, U is closed under multiplication.
2. For what pairs (a, b) of positive real numbers does the improper integral
_

b
__

x +a

x
_

x b
_
dx
70 CHAPTER 2. WILLIAM LOWELL PUTNAM COMPETITION
converge?
3. The number d
1
d
2
. . . d
9
has nine (not necessarily distinct) decimal digits. The number
e
1
e
2
. . . e
9
is such that each of the nine 9-digit numbers formed by replacing just one
of the digits d
i
is d
1
d
2
. . . d
9
by the corresponding digit e
i
(1 i 9) is divisible by 7.
The number f
1
f
2
. . . f
9
is related to e
1
e
2
. . . e
9
is the same way: that is, each of the nine
numbers formed by replacing one of the e
i
by the corresponding f
i
is divisible by 7.
Show that, for each i, d
i
f
i
is divisible by 7. [For example, if d
1
d
2
. . . d
9
= 199501996,
then e
6
may be 2 or 9, since 199502996 and 199509996 are multiples of 7.]
4. Suppose we have a necklace of n beads. Each bead is labeled with an integer and the
sum of all these labels is n 1. Prove that we can cut the necklace to form a string
whose consecutive labels x
1
, x
2
, . . . , x
n
satisfy
k

i=1
x
i
k 1 for k = 1, 2, . . . , n.
5. Let x
1
, x
2
, . . . , x
n
be dierentiable (real-valued) functions of a single variable f which
satisfy
dx
1
dt
= a
11
x
1
+a
12
x
2
+ +a
1n
x
n
dx
2
dt
= a
21
x
1
+a
22
x
2
+ +a
2n
x
n
.
.
.
.
.
.
dx
n
dt
= a
n1
x
1
+a
n2
x
2
+ +a
nn
x
n
for some constants a
ij
> 0. Suppose that for all i, x
i
(t) 0 as t . Are the
functions x
1
, x
2
, . . . , x
n
necessarily linearly dependent?
6. Suppose that each of n people writes down the numbers 1,2,3 in random order in
one column of a 3 n matrix, with all orders equally likely and with the orders for
dierent columns independent of each other. Let the row sums a, b, c of the resulting
matrix be rearranged (if necessary) so that a b c. Show that for some n 1995,
it is at least four times as likely that both b = a +1 and c = a +2 as that a = b = c.
7. For a partition of 1, 2, 3, 4, 5, 6, 7, 8, 9, let (x) be the number of elements in the
part containing x. Prove that for any two partitions and

, there are two distinct


numbers x and y in 1, 2, 3, 4, 5, 6, 7, 8, 9 such that (x) = (y) and

(x) =

(y).
[A partition of a set S is a collection of disjoint subsets (parts) whose union is S.]
2.12. 57
TH
ANUAL WILLIAM LOWELL PUTNAM COMPETITION, 1996 71
8. An ellipse, whose semi-axes have lengths a and b, rolls without slipping on the curve
y = c sin
_
x
a
_
. How are a, b, c related, given that the ellipse completes one revolution
when it traverses one period of the curve?
9. To each positive integer with n
2
decimal digits, we associate the determinant of the
matrix obtained by writing the digits in order across the rows. For example, for
n = 2, to the integer 8617 we associate det
_
8 6
1 7
_
= 50. Find, as a function of n,
the sum of all the determinants associated with n
2
-digit integers. (Leading digits are
assumed to be nonzero; for example, for n = 2, there are 9000 determinants.)
10. Evaluate
8

_
2207
1
2207
1
2207...
.
Express your answer in the form
a+b

c
d
, where a, b, c, d are integers.
11. A game starts with four heaps of beans, containing 3,4,5 and 6 beans. The two players
move alternately. A move consists of taking either
a) one bean from a heap, provided at least two beans are left behind in that heap,
or
b) a complete heap of two or three beans.
The player who takes the last heap wins. To win the game, do you want to move rst
or second? Give a winning strategy.
12. For a positive real number , dene
S() = n| : n = 1, 2, 3, . . ..
Prove that 1, 2, 3, . . . cannot be expressed as the disjoint union of three sets S(), S()
and S(). [As usual, x| is the greatest integer x.]
2.12 57
th
Anual William Lowell Putnam Competition,
1996
1. Find the least number A such that for any two squares of combined area 1, a rectangle
of area A exists such that the two squares can be packed in the rectangle (without
interior overlap). You may assume that the sides of the squares are parallel to the
sides of the rectangle.
72 CHAPTER 2. WILLIAM LOWELL PUTNAM COMPETITION
2. Let C
1
and C
2
be circles whose centers are 10 units apart, and whose radii are 1 and
3. Find, with proof, the locus of all points M for which there exists points X on C
1
and Y on C
2
such that M is the midpoint of the line segment XY .
3. Suppose that each of 20 students has made a choice of anywhere from 0 to 6 courses
from a total of 6 courses oered. Prove or disprove: there are 5 students and 2 courses
such that all 5 have chosen both courses or all 5 have chosen neither course.
4. Let S be the set of ordered triples (a, b, c) of distinct elements of a nite set A.
Suppose that
(a) (a, b, c) S if and only if (b, c, a) S;
(b) (a, b, c) S if and only if (c, b, a) / S;
(c) (a, b, c) and (c, d, a) are both in S if and only if (b, c, d) and (d, a, b) are both in
S.
Prove that there exists a one-to-one function g from A to R such that g(a) < g(b) <
g(c) implies (a, b, c) S. Note: R is the set of real numbers.
5. If p is a prime number greater than 3 and k = 2p/3|, prove that the sum
_
p
1
_
+
_
p
2
_
+ +
_
p
k
_
of binomial coecients is divisible by p
2
.
6. Let c > 0 be a constant. Give a complete description, with proof, of the set of all
continuous functions f : R R such that f(x) = f(x
2
+c) for all x R. Note that
R denotes the set of real numbers.
7. Dene a selsh set to be a set which has its own cardinality (number of elements)
as an element. Find, with proof, the number of subsets of 1, 2, . . . , n which are
minimal selsh sets, that is, selsh sets none of whose proper subsets is selsh.
8. Show that for every positive integer n,
_
2n 1
e
_
2n1
2
< 1 3 5 (2n 1) <
_
2n + 1
e
_
2n+1
2
.
9. Given that x
1
, x
2
, . . . , x
n
= 1, 2, . . . , n, nd, with proof, the largest possible value,
as a function of n (with n 2), of
x
1
x
2
+x
2
x
3
+ +x
n1
x
n
+x
n
x
1
.
2.13. 58
TH
ANUAL WILLIAM LOWELL PUTNAM COMPETITION, 1997 73
10. For any square matrix A, we can dene sin A by the usual power series:
sin A =

n=0
(1)
n
(2n + 1)!
A
2n+1
.
Prove or disprove: there exists a 2 2 matrix A with real entries such that
sin A =
_
1 1996
0 1
_
.
11. Given a nite string S of symbols X and O, we write (S) for the number of Xs in
S minus the number of Os. For example, (XOOXOOX) = 1. We call a string
S balanced if every substring T of (consecutive symbols of) S has 2 (T) 2.
Thus, XOOXOOX is not balanced, since it contains the substring OOXOO. Find,
with proof, the number of balanced strings of length n.
12. Let (a
1
, b
1
), (a
2
, b
2
), . . . , (a
n
, b
n
) be the vertices of a convex polygon which contains
the origin in its interior. Prove that there exist positive real numbers x and y such
that
(a
1
, b
1
)x
a
1
y
b
1
+ (a
2
, b
2
)x
a
2
y
b
2
+ + (a
n
, b
n
)x
an
y
bn
= (0, 0)
2.13 58
th
Anual William Lowell Putnam Competition,
1997
1. A rectangle, HOMF, has sides HO = 11 and OM = 5. A triangle ABC has H
as the intersection of the altitudes, O the center of the circumscribed circle, M the
midpoint of BC, and F the foot of the altitude from A. What is the length of BC?
2. Players 1, 2, 3, . . . , n are seated around a table, and each has a single penny. Player
1 passes a penny to player 2, who then passes two pennies to player 3. Player 3 then
passes one penny to Player 4, who passes two pennies to Player 5, and so on, players
alternately passing one penny or two to the next player who still has some pennies.
A player who runs out of pennies drops out of the game and leaves the table. Find
an innite set of numbers n for which some player ends up with all n pennies.
3. Evaluate
_

0
_
x
x
3
2
+
x
5
2 4

x
7
2 4 6
+
__
1 +
x
2
2
2
+
x
4
2
2
4
2
+
x
6
2
2
4
2
6
2
+
_
dx.
74 CHAPTER 2. WILLIAM LOWELL PUTNAM COMPETITION
4. Let G be a group with identity e and : G G a function such that
(g
1
)(g
2
)(g
3
) = (h
1
)(h
2
)(h
3
)
whenever g
1
g
2
g
3
= e = h
1
h
2
h
3
. Prove that there exists an element a G such that
(x) = a(x) is a homomorphism (i.e. (xy) = (x)(y) for all x, y G).
5. Let N
n
denote the number of ordered n-tuples of positive integers (a
1
, a
2
, . . . , a
n
) such
that 1/a
1
+ 1/a
2
+. . . + 1/a
n
= 1. Determine whether N
10
is even or odd.
6. For a positive integer n and any real number c, dene x
k
recursively by x
0
= 0,
x
1
= 1, and for k 0,
x
k+2
=
cx
k+1
(n k)x
k
k + 1
.
Fix n and then take c to be the largest value for which x
n+1
= 0. Find x
k
in terms
of n and k, 1 k n.
7. Let x denote the distance between the real number x and the nearest integer. For
each positive integer n, evaluate
F
n
=
6n1

m=1
min(
m
6n
,
m
3n
).
(Here min(a, b) denotes the minimum of a and b.)
8. Let f be a twice-dierentiable real-valued function satisfying
f(x) +f

(x) = xg(x)f

(x),
where g(x) 0 for all real x. Prove that [f(x)[ is bounded.
9. For each positive integer n, write the sum

n
m=1
1/m in the form p
n
/q
n
, where p
n
and
q
n
are relatively prime positive integers. Determine all n such that 5 does not divide
q
n
.
10. Let a
m,n
denote the coecient of x
n
in the expansion of (1 + x + x
2
)
m
. Prove that
for all [integers] k 0,
0

2k
3

i=0
(1)
i
a
ki,i
1.
11. Prove that for n 2,
n terms
..
2
2

n 1 terms
..
2
2

2
(mod n).
2.14. 59
TH
ANUAL WILLIAM LOWELL PUTNAM COMPETITION, 1998 75
12. The dissection of the 345 triangle shown below (into four congruent right triangles
similar to the original) has diameter 5/2. Find the least diameter of a dissection of
this triangle into four parts. (The diameter of a dissection is the least upper bound
of the distances between pairs of points belonging to the same part.)
2.14 59
th
Anual William Lowell Putnam Competition,
1998
1. A right circular cone has base of radius 1 and height 3. A cube is inscribed in the
cone so that one face of the cube is contained in the base of the cone. What is the
side-length of the cube?
2. Let s be any arc of the unit circle lying entirely in the rst quadrant. Let A be the
area of the region lying below s and above the x-axis and let B be the area of the
region lying to the right of the y-axis and to the left of s. Prove that A+B depends
only on the arc length, and not on the position, of s.
3. Let f be a real function on the real line with continuous third derivative. Prove that
there exists a point a such that
f(a) f

(a) f

(a) f

(a) 0.
4. Let A
1
= 0 and A
2
= 1. For n > 2, the number A
n
is dened by concatenating the
decimal expansions of A
n1
and A
n2
from left to right. For example A
3
= A
2
A
1
= 10,
A
4
= A
3
A
2
= 101, A
5
= A
4
A
3
= 10110, and so forth. Determine all n such that 11
divides A
n
.
5. Let T be a nite collection of open discs in R
2
whose union contains a set E R
2
.
Show that there is a pairwise disjoint subcollection D
1
, . . . , D
n
in T such that
E
n
j=1
3D
j
.
Here, if D is the disc of radius r and center P, then 3D is the disc of radius 3r and
center P.
6. Let A, B, C denote distinct points with integer coordinates in R
2
. Prove that if
([AB[ +[BC[)
2
< 8 [ABC] + 1
then A, B, C are three vertices of a square. Here [XY [ is the length of segment XY
and [ABC] is the area of triangle ABC.
76 CHAPTER 2. WILLIAM LOWELL PUTNAM COMPETITION
7. Find the minimum value of
(x + 1/x)
6
(x
6
+ 1/x
6
) 2
(x + 1/x)
3
+ (x
3
+ 1/x
3
)
for x > 0.
8. Given a point (a, b) with 0 < b < a, determine the minimum perimeter of a triangle
with one vertex at (a, b), one on the x-axis, and one on the line y = x. You may
assume that a triangle of minimum perimeter exists.
9. let H be the unit hemisphere (x, y, z) : x
2
+ y
2
+ z
2
= 1, z 0, C the unit circle
(x, y, 0) : x
2
+ y
2
= 1, and P the regular pentagon inscribed in C. Determine the
surface area of that portion of H lying over the planar region inside P, and write
your answer in the form Asin +Bcos , where A, B, , are real numbers.
10. Find necessary and sucient conditions on positive integers m and n so that
mn1

i=0
(1)
i/m+i/n
= 0.
11. Let N be the positive integer with 1998 decimal digits, all of them 1; that is,
N = 1111 11.
Find the thousandth digit after the decimal point of

N.
12. Prove that, for any integers a, b, c, there exists a positive integer n such that

n
3
+an
2
+bn +c
is not an integer.
2.15 60
th
Anual William Lowell Putnam Competition,
1999
1. Find polynomials f(x),g(x), and h(x), if they exist, such that for all x,
[f(x)[ [g(x)[ +h(x) =
_

_
1 if x < 1
3x + 2 if 1 x 0
2x + 2 if x > 0.
2.15. 60
TH
ANUAL WILLIAM LOWELL PUTNAM COMPETITION, 1999 77
2. Let p(x) be a polynomial that is nonnegative for all real x. Prove that for some k,
there are polynomials f
1
(x), . . . , f
k
(x) such that
p(x) =
k

j=1
(f
j
(x))
2
.
3. Consider the power series expansion
1
1 2x x
2
=

n=0
a
n
x
n
.
Prove that, for each integer n 0, there is an integer m such that
a
2
n
+a
2
n+1
= a
m
.
4. Sum the series

m=1

n=1
m
2
n
3
m
(n3
m
+m3
n
)
.
5. Prove that there is a constant C such that, if p(x) is a polynomial of degree 1999,
then
[p(0)[ C
_
1
1
[p(x)[ dx.
6. The sequence (a
n
)
n1
is dened by a
1
= 1, a
2
= 2, a
3
= 24, and, for n 4,
a
n
=
6a
2
n1
a
n3
8a
n1
a
2
n2
a
n2
a
n3
.
Show that, for all n, a
n
is an integer multiple of n.
7. Right triangle ABC has right angle at C and BAC = ; the point D is chosen on
AB so that [AC[ = [AD[ = 1; the point E is chosen on BC so that CDE = . The
perpendicular to BC at E meets AB at F. Evaluate lim
0
[EF[.
8. Let P(x) be a polynomial of degree n such that P(x) = Q(x)P

(x), where Q(x) is a


quadratic polynomial and P

(x) is the second derivative of P(x). Show that if P(x)


has at least two distinct roots then it must have n distinct roots.
9. Let A = (x, y) : 0 x, y < 1. For (x, y) A, let
S(x, y) =

1
2

m
n
2
x
m
y
n
,
78 CHAPTER 2. WILLIAM LOWELL PUTNAM COMPETITION
where the sum ranges over all pairs (m, n) of positive integers satisfying the indicated
inequalities. Evaluate
lim
(x,y)(1,1),(x,y)A
(1 xy
2
)(1 x
2
y)S(x, y).
10. Let f be a real function with a continuous third derivative such that f(x), f

(x), f

(x), f

(x)
are positive for all x. Suppose that f

(x) f(x) for all x. Show that f

(x) < 2f(x)


for all x.
11. For an integer n 3, let = 2/n. Evaluate the determinant of the n n matrix
I +A, where I is the nn identity matrix and A = (a
jk
) has entries a
jk
= cos(j+k)
for all j, k.
12. Let S be a nite set of integers, each greater than 1. Suppose that for each integer
n there is some s S such that gcd(s, n) = 1 or gcd(s, n) = s. Show that there exist
s, t S such that gcd(s, t) is prime.
2.16. 61
ST
ANUAL WILLIAM LOWELL PUTNAM COMPETITION, 2000 79
2.16 61
st
Anual William Lowell Putnam Competition,
2000
1. Let A be a positive real number. What are the possible values of

j=0
x
2
j
, given that
x
0
, x
1
, . . . are positive numbers for which

j=0
x
j
= A?
2. Prove that there exist innitely many integers n such that n, n+1, n+2 are each the
sum of the squares of two integers. [Example: 0 = 0
2
+ 0
2
, 1 = 0
2
+ 1
2
, 2 = 1
2
+ 1
2
.]
3. The octagon P
1
P
2
P
3
P
4
P
5
P
6
P
7
P
8
is inscribed in a circle, with the vertices around the
circumference in the given order. Given that the polygon P
1
P
3
P
5
P
7
is a square of
area 5, and the polygon P
2
P
4
P
6
P
8
is aectangle of area 4, nd the maximum possible
area of the octagon.
4. Show that the improper integral
lim
B
_
B
0
sin(x) sin(x
2
) dx
converges.
5. Three distinct points with integer coordinates lie in the plane on a circle of radius
r > 0. Show that two of these points are separated by a distance of at least r
1/3
.
6. Let f(x) be a polynomial with integer coecients. Dene a sequence a
0
, a
1
, . . . of
integers such that a
0
= 0 and a
n+1
= f(a
n
) for all n 0. Prove that if there exists a
positive integer m for which a
m
= 0 then either a
1
= 0 or a
2
= 0.
7. Let a
j
, b
j
, c
j
be integers for 1 j N. Assume for each j, at least one of a
j
, b
j
, c
j
is
odd. Show that there exist integers r, s, t such that ra
j
+sb
j
+tc
j
is odd for at least
4N/7 values of j, 1 j N.
8. Prove that the expression
gcd(m, n)
n
_
n
m
_
is an integer for all pairs of integers n m 1.
9. Let f(t) =

N
j=1
a
j
sin(2jt), where each a
j
is real and a
N
is not equal to 0. Let N
k
denote the number of zeroes (including multiplicities) of
d
k
f
dt
k
. Prove that
N
0
N
1
N
2
and lim
k
N
k
= 2N.
[Editorial clarication: only zeroes in [0, 1) should be counted.]
80 CHAPTER 2. WILLIAM LOWELL PUTNAM COMPETITION
10. Let f(x) be a continuous function such that f(2x
2
1) = 2xf(x) for all x. Show that
f(x) = 0 for 1 x 1.
11. Let S
0
be a nite set of positive integers. We dene nite sets S
1
, S
2
, . . . of positive
integers as follows: the integer a is in S
n+1
if and only if exactly one of a 1 or a is
in S
n
. Show that there exist innitely many integers N for which S
N
= S
0
N +a :
a S
0
.
12. Let B be a set of more than
2
n+1
n
distinct points with coordinates of the form
(1, 1, . . . , 1) in n-dimensional space with n 3. Show that there are three
distinct points in B which are the vertices of an equilateral triangle.
2.17 62
nd
Anual William Lowell Putnam Competition,
2001
1. Consider a set S and a binary operation , i.e., for each a, b S, a b S. Assume
(a b) a = b for all a, b S. Prove that a (b a) = b for all a, b S.
2. You have coins C
1
, C
2
, . . . , C
n
. For each k, C
k
is biased so that, when tossed, it has
probability 1/(2k+1) of falling heads. If the n coins are tossed, what is the probability
that the number of heads is odd? Express the answer as a rational function of n.
3. For each integer m, consider the polynomial
P
m
(x) = x
4
(2m+ 4)x
2
+ (m2)
2
.
For what values of m is P
m
(x) the product of two non-constant polynomials with
integer coecients?
4. Triangle ABC has an area 1. Points E, F, G lie, respectively, on sides BC, CA, AB
such that AE bisects BF at point R, BF bisects CG at point S, and CG bisects AE
at point T. Find the area of the triangle RST.
5. Prove that there are unique positive integers a, n such that a
n+1
(a + 1)
n
= 2001.
6. Can an arc of a parabola inside a circle of radius 1 have a length greater than 4?
7. Let n be an even positive integer. Write the numbers 1, 2, . . . , n
2
in the squares of an
n n grid so that the k-th row, from left to right, is
(k 1)n + 1, (k 1)n + 2, . . . , (k 1)n +n.
2.18. 63
RD
ANUAL WILLIAM LOWELL PUTNAM COMPETITION, 2002 81
Color the squares of the grid so that half of the squares in each row and in each column
are red and the other half are black (a checkerboard coloring is one possibility). Prove
that for each coloring, the sum of the numbers on the red squares is equal to the sum
of the numbers on the black squares.
8. Find all pairs of real numbers (x, y) satisfying the system of equations
1
x
+
1
2y
= (x
2
+ 3y
2
)(3x
2
+y
2
)
1
x

1
2y
= 2(y
4
x
4
).
9. For any positive integer n, let n) denote the closest integer to

n. Evaluate

n=1
2
n
+ 2
n
2
n
.
10. Let S denote the set of rational numbers dierent from 1, 0, 1. Dene f : S S
by f(x) = x 1/x. Prove or disprove that

n=1
f
(n)
(S) = ,
where f
(n)
denotes f composed with itself n times.
11. Let a and b be real numbers in the interval (0, 1/2), and let g be a continuous real-
valued function such that g(g(x)) = ag(x) + bx for all real x. Prove that g(x) = cx
for some constant c.
12. Assume that (a
n
)
n1
is an increasing sequence of positive real numbers such that
lima
n
/n = 0. Must there exist innitely many positive integers n such that a
ni
+
a
n+i
< 2a
n
for i = 1, 2, . . . , n 1?
2.18 63
rd
Anual William Lowell Putnam Competition,
2002
1. Let k be a xed positive integer. The n-th derivative of
1
x
k
1
has the form
Pn(x)
(x
k
1)
n+1
where P
n
(x) is a polynomial. Find P
n
(1).
2. Given any ve points on a sphere, show that some four of them must lie on a closed
hemisphere.
82 CHAPTER 2. WILLIAM LOWELL PUTNAM COMPETITION
3. Let n 2 be an integer and T
n
be the number of non-empty subsets S of 1, 2, 3, . . . , n
with the property that the average of the elements of S is an integer. Prove that T
n
n
is always even.
4. In Determinant Tic-Tac-Toe, Player 1 enters a 1 in an empty 3 3 matrix. Player
0 counters with a 0 in a vacant position, and play continues in turn until the 3 3
matrix is completed with ve 1s and four 0s. Player 0 wins if the determinant is 0
and player 1 wins otherwise. Assuming both players pursue optimal strategies, who
will win and how?
5. Dene a sequence by a
0
= 1, together with the rules a
2n+1
= a
n
and a
2n+2
= a
n
+a
n+1
for each integer n 0. Prove that every positive rational number appears in the set
_
a
n1
a
n
: n 1
_
=
_
1
1
,
1
2
,
2
1
,
1
3
,
3
2
, . . .
_
.
6. Fix an integer b 2. Let f(1) = 1, f(2) = 2, and for each n 3, dene f(n) = nf(d),
where d is the number of base-b digits of n. For which values of b does

n=1
1
f(n)
converge?
7. Shanille OKeal shoots free throws on a basketball court. She hits the rst and misses
the second, and thereafter the probability that she hits the next shot is equal to the
proportion of shots she has hit so far. What is the probability she hits exactly 50 of
her rst 100 shots?
8. Consider a polyhedron with at least ve faces such that exactly three edges emerge
from each of its vertices. Two players play the following game:
Each player, in turn, signs his or her name on a previously unsigned
face. The winner is the player who rst succeeds in signing three faces
that share a common vertex.
Show that the player who signs rst will always win by playing as well as possible.
9. Show that, for all integers n > 1,
1
2ne
<
1
e

_
1
1
n
_
n
<
1
ne
.
2.18. 63
RD
ANUAL WILLIAM LOWELL PUTNAM COMPETITION, 2002 83
10. An integer n, unknown to you, has been randomly chosen in the interval [1, 2002]
with uniform probability. Your objective is to select n in an odd number of guesses.
After each incorrect guess, you are informed whether n is higher or lower, and you
must guess an integer on your next turn among the numbers that are still feasibly
correct. Show that you have a strategy so that the chance of winning is greater than
2/3.
11. A palindrome in base b is a positive integer whose base-b digits read the same back-
wards and forwards; for example, 2002 is a 4-digit palindrome in base 10. Note that
200 is not a palindrome in base 10, but it is the 3-digit palindrome 242 in base 9, and
404 in base 7. Prove that there is an integer which is a 3-digit palindrome in base b
for at least 2002 dierent values of b.
12. Let p be a prime number. Prove that the determinant of the matrix
_
_
_
x y z
x
p
y
p
z
p
x
p
2
y
p
2
z
p
2
_
_
_
is congruent modulo p to a product of polynomials of the formax+by+cz, where a, b, c
are integers. (We say two integer polynomials are congruent modulo p if corresponding
coecients are congruent modulo p.)
Chapter 3
Asiatic Pacic Mathematical
Olympiads
3.1 1
st
Asiatic Pacic Mathematical Olympiad, 1989
1. Let x
1
, x
2
, . . . , x
n
be positive real numbers, and let
S = x
1
+ x
2
+ +x
n
.
Prove that
(1 +x
1
)(1 +x
2
) (1 +x
n
) 1 +S +
S
2
2!
+
S
3
3!
+ +
S
n
n!
.
2. Prove that the equation
6(6a
2
+ 3b
2
+c
2
) = 5n
2
has no solutions in integers except a = b = c = n = 0.
3. Let A
1
, A
2
, A
3
be three points in the plane, and for convenience, let A
4
= A
1
,
A
5
= A
2
. For n = 1, 2, and 3, suppose that B
n
is the midpoint of A
n
A
n+1
, and
suppose that C
n
is the midpoint of A
n
B
n
. Suppose that A
n
C
n+1
and B
n
A
n+2
meet
at D
n
, and that A
n
B
n+1
and C
n
A
n+2
meet at E
n
. Calculate the ratio of the area of
triangle D
1
D
2
D
3
to the area of triangle E
1
E
2
E
3
.
4. Let S be a set consisting of m pairs (a, b) of positive integers with the property that
1 a < b n. Show that there are at least
4m
(m
n
2
4
)
3n
84
3.2. 2
ND
ASIATIC PACIFIC MATHEMATICAL OLYMPIAD, 1990 85
triples (a, b, c) such that (a, b), (a, c), and (b, c) belong to S.
5. Determine all functions f from the reals to the reals for which
(1) f(x) is strictly increasing,
(2) f(x) +g(x) = 2x for all real x,
where g(x) is the composition inverse function to f(x). (Note: f and g are said to
be composition inverses if f(g(x)) = x and g(f(x)) = x for all real x.)
3.2 2
nd
Asiatic Pacic Mathematical Olympiad, 1990
1. Given triagnle ABC, let D, E, F be the midpoints of BC, AC, AB respectively and
let G be the centroid of the triangle.
For each value of BAC, how many non-similar triangles are there in which AEGF
is a cyclic quadrilateral?
2. Let a
1
, a
2
, . . . , a
n
be positive real numbers, and let S
k
be the sum of the products of
a
1
, a
2
, . . . , a
n
taken k at a time. Show that
S
k
S
nk

_
n
k
_
2
a
1
a
2
a
n
for k = 1, 2, . . . , n 1.
3. Consider all the triangles ABC which have a xed base AB and whose altitude from
C is a constant h. For which of these triangles is the product of its altitudes a
maximum?
4. A set of 1990 persons is divided into non-intersecting subsets in such a way that:
(a) No one in a subset knows all the others in the subset,
(b) Among any three persons in a subset, there are always at least two who do not
know each other, and
(c) For any two persons in a subset who do not know each other, there is exactly
one person in the same subset knowing both of them.
a Prove that within each subset, every person has the same number of acquain-
tances.
b Determine the maximum possible number of subsets.
86 CHAPTER 3. ASIATIC PACIFIC MATHEMATICAL OLYMPIADS
Note: It is understood that if a person A knows person B, then person B will know
person A; an acquaintance is someone who is known. Every person is assumed to
know ones self.
5. Show that for every integer n 6, there exists a convex hexagon which can be
dissected into exactly n congruent triangles.
3.3 3
rd
Asiatic Pacic Mathematical Olympiad, 1991
1. Let G be the centroid of triangle ABC and M be the midpoint of BC. Let X be
on AB and Y on AC such that the points X, Y , and G are collinear and XY and
BC are parallel. Suppose that XC and GB intersect at Q and Y B and GC intersect
at P. Show that triangle MPQ is similar to triangle ABC.
2. Suppose there are 997 points given in a plane. If every two points are joined by a
line segment with its midpoint coloured in red, show that there are at least 1991 red
points in the plane. Can you nd a special case with exactly 1991 red points?
3. Let a
1
, a
2
, . . . , a
n
, b
1
, b
2
, . . . , b
n
be positive real numbers such that a
1
+a
2
+ +a
n
=
b
1
+b
2
+ +b
n
. Show that
a
2
1
a
1
+b
1
+
a
2
2
a
2
+b
2
+ +
a
2
n
a
n
+b
n

a
1
+a
2
+ +a
n
2
4. During a break, n children at school sit in a circle around their teacher to play a
game. The teacher walks clockwise close to the children and hands out candies to
some of them according to the following rule. He selects one child and gives him a
candy, then he skips the next child and gives a candy to the next one, then he skips 2
and gives a candy to the next one, then he skips 3, and so on. Determine the values
of n for which eventually, perhaps after many rounds, all children will have at least
one candy each.
5. Given are two tangent circles and a point P on their common tangent perpendicular
to the lines joining their centres. Construct with ruler and compass all the circles
that are tangent to these two circles and pass through the point P.
3.4 4
th
Asiatic Pacic Mathematical Olympiad, 1992
1. A triangle with sides a, b, and c is given. Denote by s the semiperimeter, that is
3.5. 5
TH
ASIATIC PACIFIC MATHEMATICAL OLYMPIAD, 1993 87
s =
a+b+c
2
. Construct a triangle with sides s a, s b, and s c. This process is
repeated until a triangle can no longer be constructed with the side lengths given.
For which original triangles can this process be repeated indenitely?
2. In a circle C with centre O and radius r, let C
1
, C
2
be two circles with centres O
1
,
O
2
and radii r
1
, r
2
respectively, so that each circle C
i
is internally tangent to C at
A
i
and so that C
1
, C
2
are externally tangent to each other at A.Prove that the three
lines OA, O
1
A
2
, and O
2
A
1
are concurrent.
3. Let n be an integer such that n > 3. Suppose that we choose three numbers from the
set 1, 2, . . . , n. Using each of these three numbers only once and using addition,
multiplication, and parenthesis, let us form all possible combinations.
(a) Show that if we choose all three numbers greater than n/2, then thealues of
these combinations are all distinct.
(b) Let p be a prime number such that p

n. Show that the number of ways
of choosing three numbers so that the smallest one is p and the values of the
combinations are not all distinct is precisely the number of positive divisors of
p 1.
4. Determine all pairs (h, s) of positive integers with the following property: If one draws
h horizontal lines and another s lines which satisfy:
i they are not horizontal,
ii no two of them are parallel,
iii no three of the h +s lines are concurrent,
then the number of regions formed by these h +s lines is 1992.
5. Find a sequence of maximal length consisting of non-zero integers in which the sum
of any seven consecutive terms is positive and that of any eleven consecutive terms
is negative.
3.5 5
th
Asiatic Pacic Mathematical Olympiad, 1993
1. Let ABCD be a quadrilateral such that all sides have equal length and angle ABC is
60 deg. Let l be a line passing through D and not intersecting the quadrilateral (except
at D). Let E and F be the points of intersection of l with AB and BC respectively.
Let M be the point of intersection of CE and AF. Prove that CA
2
= CM CE.
88 CHAPTER 3. ASIATIC PACIFIC MATHEMATICAL OLYMPIADS
2. Find the total number of dierent integer values the function
f(x) = [x] + [2x] + [
5x
3
] + [3x] + [4x]
takes for real numbers x with 0 x 100.
3. Let
_
f(x) = a
n
x
n
+a
n1
x
n1
+ +a
0
and
g(x) = c
n+1
x
n+1
+c
n
x
n
+ +c
0
be non-zero polynomials with real coecients such that g(x) = (x +r)f(x) for some
real number r. If a = max([a
n
[, . . . , [a
0
[) and c = max([c
n+1
[, . . . , [c
0
[), prove that
a
c
n + 1.
4. Determine all positive integers n for which the equation
x
n
+ (2 +x)
n
+ (2 x)
n
= 0
has an integer as a solution.
5. Let P
1
, P
2
, . . . , P
1993
= P
0
be distinct points in the xy-plane with the following
properties:
i both coordinates of P
i
are integers, for i = 1, 2, . . . , 1993;
ii there is no point other than P
i
and P
i+1
on the line segment joining P
i
with P
i+1
whose coordinates are both integers, for i = 0, 1, . . . , 1992.
Prove that for some i, 0 i 1992, there exists a point Q with coordinates (q
x
, q
y
)
on the line segment joining P
i
with P
i+1
such that both 2q
x
and 2q
y
are odd integers.
3.6 6
th
Asiatic Pacic Mathematical Olympiad, 1994
1. Let f : R R be a function such that:
i For all x, y R,
f(x) +f(y) + 1 f(x +y) f(x) +f(y)
ii For all x [0, 1), f(0) f(x),
iii f(1) = f(1) = 1.
3.7. 7
TH
ASIATIC PACIFIC MATHEMATICAL OLYMPIAD, 1995 89
Find all such functions f.
2. Given a nondegenerate triangle ABC, with circumcentre O, orthocentre H, and
circumradius R, prove that [OH[ < 3R.
3. Let n be an integer of the form a
2
+ b
2
, where a and b are relatively prime integers
and such that if p is a prime, p

n, then p divides ab. Determine all such n.
4. Is there an innite set of points in the plane such that no three points are collinear,
and the distance between any two points is rational?
5. You are given three lists A, B, and C. List A contains the numbers of the form 10
k
in base 10, with k any integer greater than or equal to 1. Lists B and C contain the
same numbers translated into base 2 and 5 respectively:
A B C
10 1010 20
100 1100100 400
1000 1111101000 13000
.
.
.
.
.
.
.
.
.
Prove that for every integer n > 1, there is exactly one number in exactly one of the
lists B or C that has exactly n digits.
3.7 7
th
Asiatic Pacic Mathematical Olympiad, 1995
1. Determine all sequences of real numbers a
1
, a
2
, . . . , a
1995
which satisfy:
2
_
a
n
(n 1) a
n+1
(n 1), for n = 1, 2, . . . 1994,
and
2

a
1995
1994 a
1
+ 1.
2. Let a
1
, a
2
, . . . , a
n
be a sequence of integers with values between 2 and 1995 such
that:
i Any two of the a
i
s are realtively prime,
ii Each a
i
is either a prime or a product of primes.
Determine the smallest possible values of n to make sure that the sequence will contain
a prime number.
90 CHAPTER 3. ASIATIC PACIFIC MATHEMATICAL OLYMPIADS
3. Let PQRS be a cyclic quadrilateral such that the segments PQ and RS are not
parallel. Consider the set of circles through P and Q, and the set of circles through
R and S. Determine the set A of points of tangency of circles in these two sets.
4. Let C be a circle with radius R and centre O, and S a xed point in the interior of
C. Let AA

and BB

be perpendicular chords through S. Consider the rectangles


SAMB, SBN

, SA

, and SB

NA. Find the set of all points M, N

, M

, and
N when A moves around the whole circle.
5. Find the minimum positive integer k such that there exists a function f from the
set Z of all integers to 1, 2, . . . k with the property that f(x) ,= f(y) whenever
[x y[ 5, 7, 12.
3.8 8
th
Asiatic Pacic Mathematical Olympiad, 1996
1. Let ABCD be a quadrilateral AB = BC = CD = DA. Let MN and PQ be
two segments perpendicular to the diagonal BD and such that the distance between
them is d > BD/2, with M AD, N DC, P AB, and Q BC. Show that the
perimeter of hexagon AMNCQP does not depend on the position of MN and PQ
so long as the distance between them remains constant.
2. Let m and n be positive integers such that n m. Prove that
2
n
n!
(m+n)!
(mn)!
(m
2
+m)
n
3. Let P
1
, P
2
, P
3
, P
4
be four points on a circle, and let I
1
be the incentre of the triangle
P
2
P
3
P
4
; I
2
be the incentre of the triangle P
1
P
3
P
4
; I
3
be the incentre of the triangle
P
1
P
2
P
4
; I
4
be the incentre of the triangle P
1
P
2
P
3
. Prove that I
1
, I
2
, I
3
, I
4
are the
vertices of a rectangle.
4. The National Marriage Council wishes to invite n couples to form 17 discussion groups
under the following conditions:
(a) All members of a group must be of the same sex; i.e. they are either all male or
all female.
(b) The dierence in the size of any two groups is 0 or 1.
(c) All groups have at least 1 member.
(d) Each person must belong to one and only one group.
3.9. 9
TH
ASIATIC PACIFIC MATHEMATICAL OLYMPIAD, 1997 91
Find all values of n, n 1996, for which this is possible. Justify your answer.
5. Let a, b, c be the lengths of the sides of a triangle. Prove that

a +b c +

b +c a +

c +a b

a +

b +

c ,
and determine when equality occurs.
3.9 9
th
Asiatic Pacic Mathematical Olympiad, 1997
1. Given
S = 1 +
1
1 +
1
3
+
1
1 +
1
3
+
1
6
+ +
1
1 +
1
3
+
1
6
+ +
1
1993006
where the denominators contain partial sums of the sequence of reciprocals of trian-
gular numbers (i.e. k = n(n + 1)/2 for n = 1, 2, . . . , 1996). Prove that S > 1001.
2. Find an integer n, where 100 n 1997, such that
2
n
+ 2
n
is also an integer.
3. Let ABC be a triangle inscribed in a circle and let
l
a
=
m
a
M
a
, l
b
=
m
b
M
b
, l
c
=
m
c
M
c
,
where m
a
, m
b
, m
c
are the lengths of the angle bisectors (internal to the triangle) and
M
a
, M
b
, M
c
are the lengths of the angle bisectors extended until they meet the circle.
Prove that
l
a
sin
2
A
+
l
b
sin
2
B
+
l
c
sin
2
C
3,
and that equality holds i ABC is an equilateral triangle.
4. Triangle A
1
A
2
A
3
has a right angle at A
3
. A sequence of points is now dened by
the following iterative process, where n is a positive integer. From A
n
(n 3), a
perpendicular line is drawn to meet A
n2
A
n1
at A
n+1
.
(a) Prove that if this process is continued indenitely, then one and only one point
P is interior to every triangle A
n2
A
n1
A
n
, n 3.
92 CHAPTER 3. ASIATIC PACIFIC MATHEMATICAL OLYMPIADS
(b) Let A
1
and A
3
be xed points. By considering all possible locations of A
2
on
the plane, nd the locus of P.
5. Suppose that n people A
1
, A
2
, . . ., A
n
, (n 3) are seated in a circle and that A
i
has
a
i
objects such that
a
1
+a
2
+ +a
n
= nN,
where N is a positive integer. In order that each person has the same number of
objects, each person A
i
is to give or to receive a certain number of objects to or from
its two neighbours A
i1
and A
i+1
. (Here A
n+1
means A
1
and A
n
means A
0
.) How
should this redistribution be performed so that the total number of objects transferred
is minimum?
3.10 10
th
Asiatic Pacic Mathematical Olympiad, 1998
1. Let F be the set of all ntuples (A
1
, . . . , A
n
) such that each A
i
is a subset of
1, 2, . . . , 1998. Let [A[ denote the number of elements of the set A. Find:

(A
1
,...,An)F
[A
1
A
n
[
2. Show that for any positive integers a and b, (36a +b)(a +36b) can not be a power of
2.
3. Let a, b, c be positive real numbers. Pruve that:
_
1 +
a
b
_
_
1 +
b
c
_
_
1 +
c
a
_
2
_
1 +
a +b +c
3

abc
_
4. Let ABC be a triangle and D the foot of the altitude from A. Let E and F lie on
a line through D such that AE is perpendicular to BC, AF is perpendicular to CF,
and E and F are dierent from D. Let M and N be the midpoint of the segments
BC and EF, respectively. Prove that AN is perpendicular to NM.
5. Find the largest integer n such that n is divisible by all positive integers less than
3

n.
3.11. 11
TH
ASIATIC PACIFIC MATHEMATICAL OLYMPIAD, 1999 93
3.11 11
th
Asiatic Pacic Mathematical Olympiad, 1999
1. Find the smallest positive integer n with the following property: there does not exist
an arithmetic progression of 1999 real numbers containing exactly n integers.
2. Let a
1
, a
2
, . . . be a sequence of real numbers satisfying a
i+j
a
i
+ a
j
for all i, j =
1, 2, . . .. Prove that
a
1
+
a
2
2
+
a
3
3
+ +
a
n
n
a
n
for each positive integer n.
3. Let
1
and
2
be two circles intersecting at P and Q. The common tangent, closer
to P, of
1
and
2
touches
1
at A and
2
at B. The tangent of
1
at P meets
2
at
C, which is dierent from P, and the extension of AP meets BC at R. Prove that
the circumcircle of triangle PQR is tangent to BP and BR.
4. Determine all pairs (a, b) of integers with the property that the numbers a
2
+4b and
b
2
+ 4a are both perfect squares.
5. Let S be a set of 2n + 1 points in the plane such that no three are collinear and no
four concyclic. A circle will be called good if it has 3 points of S on its circumference,
n 1 points in its interior and n 1 points in its exterior. Prove that the number of
good circles has the same parity as n.
3.12 12
th
Asiatic Pacic Mathematical Olympiad, 2000
1. Compute the sum
S =
101

i=0
x
3
i
1 3x
i
+ 3x
2
i
for
x
i
=
i
101
2. Given the following triangular arrangement of circles:
94 CHAPTER 3. ASIATIC PACIFIC MATHEMATICAL OLYMPIADS

Each of the numbers 1, . . . , 9 is to be written into one of these circles, so that each
circle contain exactly one of these numbers and:
i the sum of the four numbers on each side of the triangle are equal;
ii the sum of the squares of the four numbers on each side of the triangle are equal.
Find all ways in which this can be done.
3. Let ABC be a triangle. Let M and N be the points in which the median and the
angle bisector, respectively, at A meet the side BC. Let Q and P be the point in
which the perpendicular at N to NA meets MA and BA, respectively, and O the
point in which the perpendicular at P to BA meets AN produced. Prove that QO
is perpendicular to BC.
4. Let n, k be given positive integers with n > k. Prove that
1
n + 1

n
n
k
k
(n k)
nk
<
n!
k! (n k)!
<
n
n
k
k
(n k)
nk
5. Given a permutation (a
0
, a
1
, . . . , a
n
) of the sequence 0, 1, . . . , n. A transposition of
a
i
with a
j
is called legal if a
i
= 0 for i > 0, and a
i1
+ 1 = a
j
. The permu-
tation (a
0
, . . . , a
n
) is called regular if after a number of transpositions it becomes
(1, 2, . . . , n, 0). For which numbers n is the permutation (1, n, n 1, . . . , 3, 2, 0) reg-
ular?
3.13 13
th
Asiatic Pacic Mathematical Olympiad, 2001
1. For a positive integer n let S(n) be the sum of digits in the decimal representation of
n. Any positive integer obtained by removing several (at least one) digits from the
3.14. 14
TH
ASIATIC PACIFIC MATHEMATICAL OLYMPIAD, 2002 95
right-hand end of the decimal representation of n is called a stump of n. Let T(n)
be the sum of all stumps of n. Prove that n = S(n) + 9T(n).
2. Find the largest positive integer N so that the number of integers in the set 1, 2, . . . , N
which are divisible by 3 is equal to the number of integers which are divisible by 5 or
7 (or both).
3. Let two equal regular n-gons S and T be located in the plane such that their inter-
section is a 2n-gon (n 3). The sides of the polygon S are coloured in red and the
sides of T in blue. Prove that the sum of the lengths of the blue sides of the polygon
S T is equal to the sum of the lengths of its red sides.
4. A point in the plane with a cartesian coordinate system is called a mixed point if one
of its coordinates is rational and the other one is irrational. Find all polynomials
with real coecients such that their graphs do not contain any mixed point.
5. Find the greatest integer n, such that there are n+4 points A, B, C, D, X
1
, . . . , X
n
in
the plane with AB ,= CD that satisfy the following condition: for each i = 1, 2, . . . , n
triangles ABX
i
and CDX
i
are equal.
3.14 14
th
Asiatic Pacic Mathematical Olympiad, 2002
1. Let a
1
, a
2
, a
3
, . . . , a
n
be a sequence of non-negative integers, where n is a positive
integer. Let
A
n
=
a
1
+ a
2
+ +a
n
n
.
Prove that
a
1
!a
2
! . . . a
n
! (A
n
|!)
n
,
where A
n
| is the greatest integer less than or equal to A
n
, and a! = 1 2 a
for a 1 (and 0! = 1). When does equality hold?
2. Find all positive integers a and b such that
a
2
+b
b
2
a
and
b
2
+a
a
2
b
are both integers.
3. Let ABC be an equilateral triangle. Let P be a point on the side AC and Q be a
point on the side AB so that both triangles ABP and ACQ are acute. Let R be
96 CHAPTER 3. ASIATIC PACIFIC MATHEMATICAL OLYMPIADS
the orthocentre of triangle ABP and S be the orthocentre of triangle ACQ. Let T be
the point common to the segments BP and CQ. Find all possible values of CBP
and BCQ such that triangle TRS is equilateral.
4. Let x, y, z be positive numbers such that
1
x
+
1
y
+
1
z
= 1.
Show that

x +yz +

y +zx +

z +xy

xyz +

x +

y +

z.
5. Let R denote the set of all real numbers. Find all functions f from R to R satisfying:
(i) there are only nitely many s in R such that f(s) = 0, and
(ii) f(x
4
+y) = x
3
f(x) +f(f(y)) for all x, y in R.
3.15 15
th
Asiatic Pacic Mathematical Olympiad, 2003
1. Let a, b, c, d, e, f be real numbers such that the polynomial
p(x) = x
8
4x
7
+ 7x
6
+ax
5
+bx
4
+cx
3
+dx
2
+ex +f
factorises into eight linear factors x x
i
, with x
i
> 0 for i = 1, 2, . . . , 8. Determine
all possible values of f.
2. Suppose ABCD is a square piece of cardboard with side length a. On a plane are
two parallel lines
1
and
2
, which are also a units apart. The square ABCD is placed
on the plane so that sides AB and AD intersect
1
at E and F respectively. Also,
sides CB and CD intersect
2
at G and H respectively. Let the perimeters of AEF
and CGH be m
1
and m
2
respectively. Prove that no matter how the square was
placed, m
1
+m
2
remains constant.
3. Let k 14 be an integer, and let p
k
be the largest prime number which is strictly
less than k. You may assume that p
k
3k/4. Let n be a composite integer. Prove:
(a) if n = 2p
k
, then n does not divide (n k)! ;
(b) if n > 2p
k
, then n divides (n k)!
4. Let a, b, c be the sides of a triangle, with a +b + c = 1, and let n 2 be an integer.
Show that
n

a
n
+b
n
+
n

b
n
+c
n
+
n

c
n
+a
n
< 1 +
n

2
2
.
3.15. 15
TH
ASIATIC PACIFIC MATHEMATICAL OLYMPIAD, 2003 97
5. Given two positive integers m and n, nd the smallest positive integer k such that
among any k people, either there are 2m of them who form m pairs of mutually
acquainted people or there are 2n of them forming n pairs of mutually unacquainted
people.

You might also like